Vous êtes sur la page 1sur 75

Infectious Disease Medicine:Question 1 During February, a 37-year-old male respiratory therapist is seen in your office 1 day after developing

fever, rigors, generalized muscle aches, and mild respiratory symptoms. He is otherwise in excellent health. The patient mentions that he had not received an influenza vaccination in the fall. Physical examination is normal except for a temperature of 39.4 C (103.0 F) and coryza. Influenza A has been documented in your community, and you have seen several patients with similar symptoms this week. Which of the following is most appropriate for managing this patient? A. Obtain a chest radiograph and blood cultures B. Obtain a nasopharyngeal culture for influenza and treat only if the result is positive C. Prescribe either amantadine, rimantadine, zanamivir, or oseltamivir D. Prescribe a fluoroquinolone antibiotic E. Administer an intramuscular or intravenous dose of ceftriaxone and prescribe azithromycin Infectious Disease Medicine:Question 2 A 35-year-old man was found to be HIV positive when he applied for life insurance. He is not certain when he became infected, but this could have occurred as long as 10 years ago. The patients only symptom is occasional night sweats two to three times each month. He otherwise feels well, works full time, and has never been hospitalized. Physical examination discloses small white exudative patches on his soft palate that are consistent with thrush. His CD4 cell count is 260/L, and his plasma HIV RNA viral load is 1550 copies/mL. Which of the following is the most appropriate management for this patient? A. Repeat the CD4 cell count; begin antiretroviral therapy only if the count is below 200/L B. Repeat the plasma HIV RNA determination; begin antiretroviral therapy only if the viral load is greater than 10,000 copies/mL C. Antiretroviral therapy is not indicated at this time because the patient is asymptomatic D. Begin treatment now with a three-drug antiretroviral regimen E. Begin treatment now with a two-drug antiretroviral regimen Infectious Disease Medicine:Question 3 A 67-year-old woman is admitted to the intensive care unit following urgent surgery because of bowel perforation and peritonitis. Despite having a colostomy and careful peritoneal lavage, she remains hypotensive and intubated postoperatively. She improves somewhat after ceftazidime and metronidazole are started by intravenous infusion, but on hospital day 9 she develops a new fever and recurrent hypotension. All intravenous catheters are replaced, blood cultures are obtained, and vancomycin is added to her regimen. On hospital day 11, a blood culture and a catheter tip culture are both growing an Enterococcus faecalis strain that is resistant to vancomycin but susceptible to ampicillin. Repeat blood cultures are obtained, and intravenous ampicillin is substituted for vancomycin. Three days after ampicillin is started, her fever persists, and she has developed an extensive rash. The blood cultures obtained on days 9 and 11 are all growing the same strain of E. faecalis. Which of the following should be done next? A. No change in antibiotics is needed, as the infected intravenous catheter was removed B. Begin piperacillin/tazobactam; stop all other antibiotics C. Begin quinupristin/dalfopristin; stop ampicillin D. Begin linezolid; stop ampicillin E. Stop ampicillin; resume this drug after the patient completes a desensitization protocol

Infectious Disease Medicine:Question 4 A 25-year-old man comes to the emergency department in April with fever and a cough of 2 weeks duration. HIV infection was diagnosed when he was incarcerated 4 years ago. He was recently released on parole. On physical examination, he appears thin. Temperature is 38.3 C (100.9 F), and respiration rate is 22/mm. Breath sounds are decreased in the right mid-lung field. Arterial oxygen saturation is 98% by pulse oximetry with the patient breathing room air. A chest radiograph shows a right middle lobe infiltrate, and he is hospitalized. Which of the following should be included in this patients admitting orders? A. Contact isolation B. Airborne isolation (negative pressure) C. Droplet isolation D. Standard precautions only Infectious Disease Medicine:Question 5 A 30-year-old man with HIV infection is evaluated because of a 1-week history of increasing headaches, low-grade fever, anorexia, nausea, and vomiting. He has not received any prescribed medications for the past 3 years because he believes that herbs and vitamins provide better therapy. On physical examination, the patient appears lethargic. Temperature is 38.2 C (100.8 F). He has extensive seborrheic dermatitis of the face and appears to have lost weight. His neck is supple. There are no focal neurologic findings. Hematocrit is 32%, the leukocyte count is 2500/L, and the platelet count is 150,000/L. Lumbar puncture is performed. The opening pressure is 39 cm H2O, leukocyte count is 25/L, protein is 65 mg/dL, and glucose is 50 mg/dL (simultaneous plasma glucose is 95 mg/dL). An India ink preparation shows many encapsulated budding yeasts, and the cryptococcal antigen titer is greater than 1:8192. In addition to beginning antifungal therapy, which of the following should be done next? A. No additional therapy is indicated B. Request that a neurosurgeon place a ventriculoperitoneal shunt as quickly as possible C. Perform lumbar punctures daily D. Give dexamethasone, 8 mg daily for 1 week E. Begin highly active antiretroviral therapy Infectious Disease Medicine:Question 6 A 51-year-old male accountant is being treated for his third episode of pneumococcal pneumonia in the last 15 months. Prior to the first episode, he had never had a major illness and had never been hospitalized. There have been no significant personal or occupational changes and no history of recent travel. His children are grown and live away from home. Physical examination and a chest radiograph are consistent with right lower lobe pneumonia. A CT scan of the chest shows no endobronchial or other mass lesions. The leukocyte count is 14,500/L with a slight left shift. Determination of lymphocyte subsets shows a normal ratio of CD4/CD8 cells. Total hemolytic complement (CH5O) determination is normal. Serum protein electrophoresis shows gamma globulins in the low-normal range without a monoclonal spike. Which of the following disorders is the most likely cause of this patients multiple episodes of pneumonia? A. Terminal complement component deficiency B. Bronchogenic carcinoma C. Common variable hypogammaglobulinemia

D. Multiple myeloma

Infectious Disease Medicine:Question 7 A 75-year-old female nursing-home resident is brought to the emergency department because of a 2-day history of generalized weakness and fever. The patient requires a chronic indwelling urinary catheter. On physical examination, temperature is 38.4C (101.1 F), pulse rate is 95/min, respiration rate is 22/mm, and blood pressure is 132/72 mm Hg. Examination findings are otherwise normal for a patient of her age. The leukocyte count is 13,000/L (with 11% immature band forms). Urinalysis shows 20-25 leukocytes/hpf. A complete metabolic profile is normal. Arterial oxygen saturation is 95% by pulse oximetry with the patient breathing room air. Urine culture and two sets of blood cultures obtained in the emergency department are growing Escherichia coli. Which of the following terms best describes this patients illness? A. Systemic inflammatory response syndrome B. Septic shock C. Bacteremia D. Sepsis E. Severe sepsis Infectious Disease Medicine:Question 8 A 54-year-old man is evaluated because of fatigue, backache, and intermittent fever of 3 months duration. He has no history of cardiac disease or drug allergies. On physical examination, there are three splinter hemorrhages under his fingernails but no other abnormalities of his skin. Ophthalmologic examination reveals a right conjunctival hemorrhage. Funduscopic examination is normal. The lungs are clear. Cardiac examination discloses a soft diastolic murmur of aortic insufficiency, which is a new finding. There is no splenomegaly. Neurologic examination is normal. Four sets of blood cultures grow a microorganism of the viridans streptococci group, which is sensitive to penicillin. A transthoracic echocardiogram shows a thickened bicuspid aortic valve with evidence of mild aortic insufficiency. A transesophageal echocardiogram confirms these findings and also shows an oscillating mass on the aortic valve. Which of the following intravenous agents is the most appropriate initial antibiotic therapy for this patient? A. Vancomycin for 4 weeks B. Penicillin G for 4 weeks C. Penicillin G plus gentamicin, both for 4 weeks D. Penicillin G plus gentamicin, both for 6 weeks E. Ceftriaxone for 8 weeks Infectious Disease Medicine:Question 9 Two days ago, a previously healthy 27-year-old man came to the emergency department because of a 2-day history of fever, headache, and general malaise. Other than a temperature of 37.9 C (101.2 F), physical examination was normal. Complete blood count, serum electrolytes, blood urea nitrogen, serum creatinine, CT scan of the head, and cerebrospinal fluid examination were normal. The patient was given an antipyretic agent and sent home with instructions to rest and drink sufficient fluids. The next day, he returned to the emergency department because of new skin lesions on his face and neck. These measured 2 to 3 mm in diameter and were erythematous and papular with a slight tendency to becoming vesicular. Chickenpox was diagnosed on clinical grounds. No additional laboratory tests were done, no medications were prescribed, and he was sent home again. Today, the patient returns to the emergency department because his general symptoms and the facial rash are worse, and more

lesions are present on his face, arms, and legs. The original facial lesions now measure 5 to 6 mm in diameter and are frankly pustular, and the lesions on his arms and legs resemble the facial lesions seen yesterday.

Which of the following prophylactic measures should be done first for all persons (health care workers, family members, friends, and coworkers) who had close contact with the patient? A. Administer vaccinia immune globulin B. Administer ribavirin C. Administer vaccinia vaccine D. Administer vaccinia vaccine concurrently with vaccinia immune globulin E. Send all health care workers who had contact with the patient home from day 6 to day 18 after the exposure Infectious Disease Medicine:Question 10 A 45-year-old woman is planning a 2-week trip to Kenya. Her itinerary includes overnight stays in several game parks. She has mild osteoarthritis, no known allergies, and takes no prescription medications. She seeks your advice regarding prevention of malaria. Which of the following is indicated for prophylaxis of malaria in this patient? A. Chloroquine B. Quinine C. Atovaquone/proguanil (Malarone ) D. Metronidazole E. Trimethoprim/sulfamethoxazole

Infectious Disease Medicine: Question 11 A 19-year-old man is evaluated after his seventh episode of bacterial sinusitis in the last 10 years. He has also had two episodes of bacterial pneumonia during the same time period as well as probable bacterial pneumonia when he was 7 years old. Quantitative serum immunoglobulin determination shows that the IgA is below the limits of detection and that the lgG and 1gM are both in the high-normal range. The patient is treated with oral amoxicillin and has a prompt response. Which of the following diagnostic studies should be ordered next to help define the reason for the patients multiple bacterial infections? A. Measurement of total hemolytic complement (CH50) B. Lymphocyte subset quantification C. Repeat quantitative serum immunoglobulin determination D. Serum IgA subset quantification E. Serum lgG subset quantification

Infectious Disease Medicine:Question 12 A 23-year-old man was seen at a local clinic for sexually transmitted diseases because of his second episode of symptomatic primary genital syphilis. He asked why this happened on two different occasions. He has been sexually active since 15 years of age, and most of his contacts have been with other men. He usually uses condoms but reports that they sometimes tear. His last HIV test 9 months ago was negative. Which of the following is the best explanation for his repeated episodes of syphilis? A. Occult HIV infection B. Failure to treat occult Chlamydia infection C. Sexual contacts within a partner network with a high prevalence of syphilis D. Incomplete treatment of his previous episode of syphilis

E. Infection with penicillin-resistant Treponema pallidum

Infectious Disease Medicine:Question 13 A 32-year-old woman from New Jersey is evaluated because of a 3-day history of a slowly expanding lesion on her left thigh that is not painful or itching. The patient removed a tick from the site of the lesion approximately 3 weeks ago. She has documented allergies to doxycycline, sulfa drugs, and codeine. On physical examination, she has a 10-cm ovoid erythematous lesion with no central clearing on her thigh. Examination is otherwise unremarkable. An antibody assay for Borrelia burgdorferi is negative. Which of the following is most appropriate for treating this patient at this time? A. Defer treatment pending the results of a repeat B. burgdorferi assay in 6 weeks B. Administer amoxicillin for 21 days C. Administer doxycycline for 21 days D. Administer azithromycin for 21 days Infectious Disease Medicine:Question 14 A 36-year-old man who has been HIV positive for approximately 10 years is brought to the emergency department after a witnessed seizure. He had been receiving antiretroviral medications until approximately 5 years ago, when he dropped out of care. The patient has taken no medications since that time and has no history of AIDS-related problems. Family members report that he has had some memory loss and unusual behavior for the past 2 weeks. On physical examination, he is confused and disoriented but does not appear to be chronically ill. Vital signs are normal. An MRI scan of the head shows a single ringenhancing lesion within the left cerebral hemisphere arising from the basal ganglia with significant mass effect, including midline shift. The patient is hospitalized, and high-dose dexamethasone is begun. The following day, his CD4 cell count is 17/L. Toxoplasma 1gM titer is negative, andToxoplasma lgG titer is positive. Cytomegalovirus 1gM titer is negative, and cytomegalovirus lgG titer is positive. Which of the following is most appropriate for initial management of this patient? A. Treat empirically for toxoplasmic encephalitis B. Order stereotactic biopsy of the brain lesion C. Perform lumbar puncture for cerebrospinal fluid culture and cytologic studies D. Treat empirically for cytomegalovirus encephalitis E. Treat empirically for bacterial brain abscess Infectious Disease Medicine:Question 15 A 68-year-old woman is transferred to your institution from an outlying hospital because of worsening pneumonia and respiratory failure. She had been in the other hospital for 11 days following a cerebrovascular accident and a nosocomial urinary tract infection that had been treated with ceftazidime. Her transfer records are incomplete but note that a tracheal aspirate 2 days ago showed copious leukocytes and very resistantAcinetobacter baumannii. A chest radiograph at the time of transfer shows two areas of consolidation in the right lung. These findings are interpreted as being compatible with a hospitalacquired pneumonia. Emergency Gram stain of sputum shows numerous leukocytes and gram-negative coccobacillary forms. Which of the following is the most reasonable choice of antibiotics for this patient until further information is available? A. Imipenem

B. Cefepime C. Levofloxacin D. Ertapenem E. Gentamicin

Infectious Disease Medicine:Question 16 At a routine office visit, a 42-year-old woman mentions that her episodes of recurrent genital herpes have become more frequent. Your records show that she received treatment for five episodes in the last 11 months, and she states that she did not even seek therapy for three episodes. She asks if anything can be done to prevent these episodes. Which of the following treatment regimens will most likely reduce the number of episodes of genital herpes in this patient over the next year? A. Topical acyclovir daily for 1 month B. Intravenous acyclovir daily for 21 days C. Oral acyclovir, valacyclovir, or famciclovir daily for 21 days D. Oral prednisone plus oral acyclovir daily for 1 week every month E. Oral acyclovir, famciclovir, or valacyclovir daily to be taken indefinitely Infectious Disease Medicine: Question 17 An 18-year-old female college student is brought to the emergency department by her friends because of altered mental status and fever. She was well until several hours ago, when she began complaining of a headache. On physical examination, she is obtunded. Temperature is 39.0 C (102.2 F). She is tachycardic and hypotensive and has multiple petechiae and some purpura over her lower extremities. Her lungs are clear. Abdominal examination is normal. An emergency department intern intubates the patient for airway protection. The intern did not have time to put on a mask while doing this procedure. A nurse, who is wearing only gloves, starts peripheral intravenous access for hydration and antibiotics. A radiology technician obtains a portable chest film. The chaplain visits briefly in the patients room. Following these procedures, lumbar puncture is performed, which reveals Neisseria meningitidis. Which of the following hospital personnel requires meningococcal post-exposure prophylaxis? A. The nurse B. The radiology technician C. The intern D. The chaplain E. All of the above Infectious Disease Medicine:Question 18 A 65-year-old retired male construction engineer spent 1 month helping to build houses on an Indian reservation in southern Arizona. Six weeks after his return to Georgia, he consults you because of fever, cough productive of minimal sputum, fatigue, and mild dyspnea on exertion. These symptoms have been present for about 3 weeks and are not improving with levofloxacin prescribed by his family physician. Physical examination is normal. Laboratory studies are normal except for a hematocrit of 35% and an erythrocyte sedimentation rate of 65 mm/h. A chest radiograph shows patchy pulmonary infiltrates in the right middle and right lower lobes. Which of the following is the most likely diagnosis? A. Hantavirus pulmonary syndrome

B. Nocardiosis C. Atypical mycobacterial infection D. Coccidioidomycosis E. Ehrlichiosis

Infectious Disease Medicine:Question 19 A 45-year-old woman is brought to the emergency department after she becomes unresponsive. Family members state that she developed an earache 3 days ago. Her primary care physician diagnosed otitis media and prescribed ciprofloxacin. However, the patient did not improve and became increasingly lethargic. Medical history is noncontributory, she has no allergies, and her only medication is ciprofloxacin. On physical examination, the patient is obtunded and has meningismus. Temperature is 40.0 C (104.0F), pulse rate is 120/ min, respiration rate is 32/mm, and blood pressure is 80/50 mm Hg. A purpuric rash is present on her lower extremities. The leukocyte count is 25,000/L (with 25% band forms), and the platelet count is 20,000/L. A lumbar puncture is performed. The cerebrospinal fluid is cloudy. The leukocyte count is 2500/L (with 99% neutrophils), glucose is 20 mg/dL (simultaneous plasma glucose is 72 mg/dL), and protein is 230 mg/dL. A Gram stain of cerebrospinal fluid shows many neutrophils and gram-positive diplococci in pairs. Which of the following empiric antimicrobial regimens is most appropriate? A. Penicillin B. Ceftriaxone C. Vancomycin D. Vancomycin plus ceftriaxone E. Vancomycin plus ampicillin

Infectious Disease Medicine:Question 20 A 34-year-old female health care worker had a tuberculin skin test done as part of her annual physical examination. The result showed a 17-mm induration. Three previous skin tests were negative with no palpable induration. She was advised to take isoniazid prophylaxis. However, after a discussion with her physician, she decided not to take the drug because of concerns about potential hepatotoxicity. The patient was especially concerned about drug side effects because she had recently learned that she has mild multiple sclerosis that does not require treatment. One month after her annual physical examination, she develops cough and fever. A chest radiograph shows a hazy density in the posterior left upper lobe, and azithromycin is prescribed. When seen 1 week later, she reports only slight improvement. A repeat chest radiograph shows no change, and the density may even have increased. Which of the following should be done next? A. Order a swallowing evaluation B. Change the azithromycin to levofloxacin C. Order a sputum smear and culture for acid-fast bacilli D. Repeat the tuberculin skin test

Infectious Disease Medicine: Question 21 A 25-year-old woman who has been documented to be HIV positive for 2 years is in the eighth week of her first pregnancy. She is asymptomatic. CD4 cell counts have consistently been greater than 700/L, and plasma HIV RNA viral loads have been undetectable. The patient has never taken antiretroviral medications.

Which of the following is the most appropriate management during her pregnancy? A. Begin a three-drug antiretroviral regimen, including efavirenz, now B. Begin a three-drug antiretroviral regimen, including zidovudine, now C. Antiretroviral therapy is not needed because of the undetectable HIV RNA viral load D. Administer nevirapine as a single drug at the time of delivery E. Begin zidovudine as a single drug during her second trimester

Infectious Disease Medicine:Question 22 An 18-year-old male high school basketball player came to the emergency department in February because of a red patch on his left forearm. He had been well the day before, but woke up with a painful area measuring about 6 X 9 cm on the volar surface of the forearm. The area was tender to touch, erythematous, and raised but was not fluctuant. The emergency department physician did not believe that incision and drainage were required and prescribed warm packs to the area and a course of dicloxacillin. The patient returns to the emergency department 2 days later. The patch is larger and more tender but is still not fluctuant. He is slightly ill but does not appear toxic and is able to go to school and attend basketball practice. The emergency department physician changes the antibiotic to cephalexin, but the patient continues to become somewhat worse over the next 2 days. Which of the following is the most likely cause of this patients clinical deterioration? A. Lyme disease B. An abscess C. Fasciitis D. A -lactam-resistant organism Infectious Disease Medicine:Question 23 A 58-year-old man is brought to your office by family members because he has been acting childish for 2 days. Today, he spent several hours changing the channels on the remote control, even though the television set was turned off. On physical examination, temperature is 38.4C (101.1 F). He is confused about where he is and what occurred today. There is no meningismus. Funduscopic examination and the remainder of the neurologic examination are normal, as is the general physical examination. Plasma glucose and serum electrolyte determinations are normal. CT scan of the head without contrast shows no abnormalities. Lumbar puncture is performed. Cerebrospinal fluid shows 84 leukocytes/L (93% lymphocytes and 7% neutrophils), protein of 90 mg/dL, and glucose of 75 mg/dL (simultaneous plasma glucose is 88 mg/dL). Which of the following is most appropriate at this time? A. Begin acyclovir, 5 mg/kg intravenously every 8 hours B. Begin acyclovir, 10 mg/kg intravenously every 8 hours C. Await the results of a polymerase chain reaction test of CSF before beginning antiviral therapy D. Await the results of an MRI scan of the head before beginning antiviral therapy E. Await the results of a brain biopsy before beginning antiviral therapy Infectious Disease Medicine:Question 24 A 19-year-old woman from Indianapolis comes to the emergency department of a New Jersey hospital with a tick on her right arm. She first noted the tick 9 hours ago. The patient was on a 2-week college industrial-ecologic field trip, during which time she toured a factory or a nature habitat each day and returned to the hotel each evening. She showered daily. She is healthy and takes no medications. The emergency department physician identified the tick as an Ixodes scapularis nymph

that was not engorged. After removing the tick, there was no evidence of redness or inflammation at the site of the bite, although the tick had definitely been attached. No residual tick parts were left in the skin. Which of the following is the most appropriate management at this time? A. Observation B. Amoxicillin, 500 mg orally three times daily for 10 days C. Azithromycin, 1 g orally daily for 10 days D. Cefuroxime-axetil, 500mg orally twice daily for 10 days E. Doxycycline, 200 mg orally once daily Infectious Disease Medicine:Question 25 A 33-year-old woman was hospitalized following her third episode of meningococcal meningitis in the last 20 years. She responded promptly to intravenous ceftriaxone and had no sequelae. During the acute phase of her illness and again 2 weeks after recovery, her total hemolytic complement (CH5O) values were in the mid-normal range. Which of the following diagnostic studies should be ordered next? A. No additional studies are needed B. Measurement of alternative pathway complement components C. Measurement of individual serum C5, 6, 7, and 8 levels D. Serum lgG subset quantification E. T-cell subset quantification Infectious Disease Medicine:Question 26 An 80-year-old man is hospitalized because of headache, fever, and altered sensorium. He was in his usual state of health until 2 days prior to admission when he developed a frontal headache that kept him awake that night. The next day, he felt feverish and told his wife that his neck was aching. Acetaminophen provided some relief. The following morning, his wife was unable to arouse him from sleep, and he was brought to the emergency department. On physical examination in the emergency department, temperature is 38.9C (102.0 F), pulse rate is 130/ min, respiration rate is 40/mm, and blood pressure is 110/50 mm Hg. He is obtunded and has a stiff neck. Lumbar puncture is performed. The opening pressure is high. Leukocyte count is 1200/L. (with 95% neutrophils), glucose is 22 mg/dL (simultaneous plasma glucose is 85 mg/dL), and protein is 200 mg/dL. Gram stain of cerebrospinal fluid is negative. Empiric antimicrobial therapy with vancomycin, ampicillin, and ceftriaxone is begun. Which of the following statements is correct regarding administration of adjunctive dexamethasone to this patient? A. Dexamethasone should be administered within 1 hour of the first dose of antimicrobial therapy B. Dexamethasone has no role in the adjunctive treatment of bacterial meningitis in an adult patient C. Dexamethasone will increase the risk of an unfavorable outcome in an adult patient with bacterial meningitis D. If this patient has pneumococcal meningitis, dexamethasone will improve his chance of survival E. If this patient hasListeria meningitis, dexamethasone will improve his chance of survival Infectious Disease Medicine:Question 27 A 22-year-old man was admitted to the intensive care unit in a comatose state following a motor vehicle accident. Intubation and mechanical ventilation were begun. After 1 week, he remained comatose and on mechanical ventilation and had also developed fever and purulent sputum. A chest radiograph showed a pulmonary infiltrate. A Gram-stained tracheal aspirate showed many leukocytes and gram-negative bacilli, and

culture of the aspirate grew Enterobacter cloacae that was sensitive to third-generation cephalosporins, piperacillin, and trimethoprim/sulfamethoxazole. Intravenous ceftazidime was begun. Today, 10 days after starting ceftazidime, the pulmonary infiltrates are less extensive. However, the patient remains febrile. Nurses report no diarrhea. Physical examination findings are unchanged. Blood cultures, urinalysis, and urine culture are negative. Repeat Gram stain of a tracheal aspirate shows many leukocytes and gram-negative bacilli, and repeat culture of the aspirate grows E. cloacae that is now sensitive to trimethoprim/sulfamethoxazole and resistant to all -lactams except imipenem. In addition to discontinuing ceftazidime, which of the following should be done next? A. Start imipenem to complete a 2- to 3-week course of antibiotic therapy B. Start trimethoprim/sulfamethoxazole to complete a 2- to 3-week course of antibiotic therapy C. No new antibiotics are required D. Start oral cefixime to complete a 3-week course of antibiotic therapy E. Start metronidazole to complete a 3-week course of antibiotic therapy Infectious Disease Medicine:Question 28 A 45-year-old man with known HIV infection for many years is transferred from a community hospital for additional management of pneumonia. He is a migrant worker who was born in Mexico but spent most of his life in rural Georgia. The patient has not received antiretroviral therapy and has no history of AIDS-related complications. An admission chest radiograph shows bilateral interstitial pulmonary infiltrates consistent with Pneumocystis carinii pneumonia. Admission arterial blood PO2 is 42 mm Hg with the patient breathing room air. Intubation and mechanical ventilation are required shortly after transfer. He is treated with intravenous trimethoprim/sulfamethoxazole and oral prednisone in standard doses, but his clinical condition deteriorates and hypoxemia worsens. An induced sputum specimen for P. cariniiis negative by direct fluorescent antibody stain, but the specimen was of poor quality. Bronchoscopy is performed, and microscopic examination of bronchoalveolar lavage fluid is shown. Which of the following pathogens is most likely causing this patients current findings?

A. Ascaris lumbricoides B. Pneumocystis carinii C. Taenia solium D. Strongyloides stercoralis E. Cryptosporidium parvum Infectious Disease Medicine:Question 29 In January, a 56-year-old woman with chronic obstructive pulmonary disease and type 2 diabetes mellitus comes for a routine office visit. She is currently clinically stable and has no new or acute symptoms. An outbreak of influenza A is occurring in your community, but the patient failed to receive an influenza vaccination last fall. Which of the following is most appropriate for preventing influenza in this patient?

A. Administer influenza vaccine and prescribe no new drugs B. Obtain a nasopharyngeal culture for influenza and treat only if the result is positive C. Administer influenza vaccine and prescribe amantadine, rimantadine, or oseltamivir for 2 weeks D. Administer influenza vaccine and prescribe amantadine, rimantadine, or oseltamivir for 6 weeks E. Tell the patient that it is too late for an influenza vaccination but prescribe amantadine, rimantadine, or oseltamivir for 2 weeks Infectious Disease Medicine:Question 30 A 35-year-old man was admitted to the intensive care unit (ICU) because of injuries sustained as a result of a gunshot wound. He underwent repair of a perforated colon, required hemodialysis for acute renal failure, and was treated with imipenem for Enterobacter pneumonia. On the 22nd day in the ICU, these problems are resolving, but he develops fever to 39.7 C (103.4 F) without localizing signs. All catheters are changed, blood cultures are obtained, and imipenem is restarted. On the 23rd day in the ICU, his temperature is normal, a chest radiograph shows no new pulmonary infiltrates, and the renal failure continues to resolve. On the 26th day, as he is about to be transferred from the ICU, the laboratory reports that a yeast is growing in one bottle of each of two sets of blood cultures that were obtained on the day that he became febrile. Which of the following is the most appropriate next step in managing this patient? A. Repeat the blood cultures; await culture results before beginning fluconazole B. Repeat the blood cultures; begin fluconazole immediately after specimens are obtained C. Do a funduscopic examination; begin fluconazole only if chorioretinitis is present D. Removing the catheters is adequate; no additional diagnostic studies or treatment is indicated Infectious Disease Medicine:Question 31 A 20-year-old male college student is evaluated on Tuesday morning because of a 2-day history of urethral discharge and dysuria. Gram stain and smear of a thick discharge show copious leukocytes and numerous gram-negative intracellular diplococci, and treatment for gonorrhea and possible occult Chlamydia infection is begun. In addition to advice about barrier precautions, which of the following should be discussed as a way to prevent future sexually transmitted diseases in this patient? A. Using caution about alcohol consumption B. Asking sexual partners if they might have gonorrhea C. Showering shortly after sex D. Taking prophylactic antibiotics after sex Infectious Disease Medicine:Question 32 A 49-year-old woman has had four urinary tract infections in the past 7 months. She was referred to a urologist, who found no abnormalities during cystoscopy. The patient refused further work-up. Which of the following is the most appropriate management strategy to prevent recurrent urinary tract infections in this patient? A. Long-term low-dose amoxicillin, 250mg orally daily B. Long-term full-dose trimethoprim/sulfamethoxazole, 160/800 mg orally twice daily 0. Patient-initiated 3-day course of ciprofloxacin, 250 mg orally twice daily D. Use of spermicides E. Prompt urination after sexual intercourse Infectious Disease Medicine:Question 33 A 63-year-old woman is hospitalized because of refractory cellulitis. An intravenous

cephalosporin is started, and she appears to be improving. On the fifth hospital day, however, she develops profuse diarrhea and a low-grade fever. Her abdomen is soft, with minimal diffuse tenderness to deep palpation and normal bowel sounds. Which of the following is the most appropriate next step in managing this patient? A. Obtain routine bacterial stool cultures B. Start an antimotility agent C. Institute contact isolation D. Obtain CT scans of the abdomen E. Add an aminoglycoside to her current regimen Infectious Disease Medicine:Question 34 A 78-year-old man is admitted to the intensive care unit because of severe congestive heart failure that requires monitoring with a central venous catheter. After 2 days, he develops diaphoresis, tachycardia, and a temperature of 39.5 C (103.1 F). A small amount of purulent material is noted at the catheter site. The catheter is removed, blood culture specimens are drawn, and empiric vancomycin is begun. The patient improves, but the catheter tip and both sets of blood cultures grow Staphylococcus aureus that is reported to be sensitive to oxacillin. Vancomycin is changed to nafcillin, 2 g intravenously every 4 hours. No signs of endocarditis are noted on physical examination, and repeat blood cultures show no growth. On hospital day 7, the patient appears ready for discharge, based on cardiac and hemodynamic parameters. Which of the following is the most appropriate management at this time? A. No additional diagnostic studies are needed; stop the intravenous nafcillin B. No additional diagnostic studies are needed; continue the intravenous nafcillin for a total of 8 weeks C. No additional diagnostic studies are needed; substitute oral dicloxacillin for the intravenous nafcillin for a total of 10 days of antibiotics D. Obtain a transesophageal echocardiogram; if this does not show signs of endocarditis, continue the intravenous nafcillin for a total of 14 days E. Obtain a transesophageal echocardiogram; if this does not show signs of endocarditis, continue the intravenous nafcillin for a total of 6 weeks Infectious Disease Medicine:Question 35 A 74-year-old woman is hospitalized because of chills, fever, flank pain and tenderness, dysuria, and urinary urgency and frequency. Urinalysis shows pyuria, and a urine culture grows Escherichia coli that is sensitive to ceftriaxone but is resistant to ceftazidime, fluoroquinolones, and trimethoprim/sulfamethoxazole. The patient is treated with ceftriaxone for 10 days but remains febrile. Because of persistent fever, she is transferred to another hospital 2 weeks later. Urine culture obtained at the second hospital again grows E. ccli that is sensitive to ceftriaxone, cefepime, imipenem, and piperacillin/tazobactam but is resistant to ceftazidime, fluoroquinolones, and trimethoprim/sulfamethoxazole. CT scan of the abdomen and pelvis shows no renal or perirenal abscesses, nephrolithiasis, or urinary tract obstruction. Which of the following is most appropriate regarding this patients antibiotic therapy? A. Continue ceftriaxone B. Change to aztreonam C. Change to imipenem D. Discontinue all antibiotic therapy Infectious Disease Medicine:Question 36 A 29-year-old man was found to have late-stage HIV infection at the time of diagnosis of

pulmonary tuberculosis. His initial CD4 cell count was 17/L with a plasma HIV RNA viral load of 33,520 copies/mL. Cough and fever quickly improved following treatment with a three-drug antituberculous regimen (isoniazid, rifampin, and pyrazinamide) and a threedrug antiretroviral regimen (zidovudine, lamivudine, and abacavir). At a follow-up visit 4 weeks later, he reports recurrent fevers as well as neck pain and swelling. On physical examination, temperature is 38.4C (101.1 F). Other vital signs are normal. His weight is unchanged. Examination of the neck discloses bilateral enlarged, tender, fluctuant lymph nodes. The spleen tip is palpable. A chest radiograph shows a new pleural effusion. Lymph node aspirate reveals no organisms on acid-fast stain. Which of the following is the most appropriate management at this time? A. Add ethambutol to the antituberculous regimen B. Substitute efavirenz for abacavir in the antiretroviral regimen C. Obtain an excisional lymph node biopsy D. Perform a diagnostic thoracentesis E. Treat symptomatically for pain and fever Infectious Disease Medicine:Question 37 A 43-year-old male farmer comes to your office because he developed a blister at the vermilion border of his lower lip yesterday morning. He has a long history of similar lesions, which he refers to as cold sores. He mentions that the lesions often appear after extended periods in the sun. The patient denies fever or fatigue and otherwise feels well. Physical examination is normal except for one blister on his lip. Which of the following is most appropriate at this time? A. Obtain a virus culture of the lip lesion B. Prescribe a topical antibacterial ointment to be started today C. Prescribe a 7-day course of oral acyclovir to be started today D. Prescribe topical penciclovir to be started today E. Prescribe no treatment now; discuss the option of immediate self-treatment with topical penciclovir for future episodes

Infectious Disease Medicine:Question 38 A 53-year-old female reporter is evaluated because of fever, headache, hypotension, and rash. She has just returned from Africa, where she covered a story about a small outbreak of a hemorrhagic fever. She had direct contact with two of the patients and their families, and she had spent time in their homes and in the fields where they worked. On physical examination, temperature is 38.6 C (101 .5 F), and blood pressure is 92/68 mm Hg. A petechial and purpuric rash is present on her trunk and extremities. Laboratory studies: Hemoglobin 8.5 g/dL Leukocyte count 2300/L (with a marked left shift) Platelet count 29,000/L Blood urea nitrogen 55 mgldL Serum creatinine2.9 mg/dL Serum electrolytes Normal Urinalysis Myriad erythrocytes/hpf; moderate protein; otherwise normal Because of the platelet count and bleeding into the skin and urinary tract, a lumbar puncture was not done. Which of the following organisms is least likely to be the etiologic agent causing this patients illness? A. Neisseria meningitidis B. Yersinia pestis

C. Ebola virus D. Marburg virus

Infectious Disease Medicine:Question 39 A 17-year-old male adolescent is hospitalized because of a pyogenic liver abscess. He has had numerous episodes of bacterial pneumonia, sinusitis, skin infections, and other infections for most of his life. Results of previous studies for immunoglobulin and neutrophil disorders were normal. A total hemolytic complement (CH50) value was 3 standard deviations below the mean. Which of the following laboratory studies is most appropriate to attempt to identify the cause of this patients multiple infections? A. C3 B. Properdin C. C5 D. IgA E. C1 inhibitor (C1 INH) Infectious Disease Medicine:Question 40 A 25-year-old man is hospitalized because of right lower lobe pneumonia. Blood and sputum cultures growStreptococcus pneumoniae that is sensitive to levofloxacin (minimum inhibitory concentration, 2 g/mL) but is resistant to erythromycin and penicillin (minimum inhibitory concentration, 4 g/mL). Intravenous levofloxacin is begun. However, the patient remains febrile and hypotensive. On hospital day 5, he develops a petechial rash. Blood cultures drawn on day 5 again grow gram-positive diplococci. In addition to discontinuing levofloxacin, which of the following is most appropriate at this time? A. Begin intravenous vancomycin B. Begin clindamycin C. Begin quinupristin/dalfopristin D. Begin moxifloxacin E. Begin doxycycline Infectious Disease Medicine:Question 41 A 47-year-old man with longstanding HIV infection has a 6-month history of increasing abdominal girth. HIV infection was diagnosed in 1995, and treatment with a protease inhibitor-containing antiretroviral regimen was started 2 years ago. Since beginning this regimen, the patient has had consistently undetectable plasma HIV RNA viral loads. His CD4 cell count has increased from 150/L to 440/L when last measured, and his weight has decreased from 95 kg (209 Ib) to 90 kg (198 Ib) over the same period. He feels well and is working full time. Physical examination discloses a small dorsocervical fat pad, wasting in all four extremities, and a protruding abdomen with mild hepatomegaly and some faint striae. Which of the following is most likely causing this patients current findings? A. High-grade B-cell lymphoma B. Cushings syndrome C. HIV lipodystrophy D. HIV wasting

E. Chronic hepatitis C infection Infectious Disease Medicine:Question 42 A 52-year-old woman from Kentucky has acute myeloid leukemia. After several cycles of induction chemotherapy and subsequent relapse, she undergoes matched unrelated allogeneic stem cell transplantation. Her course is complicated by Pseudomonas pneumonia and Clostridium difficile colitis prior to engraftment, acute graft-versus-host disease treated with corticosteroids and tacrolimus, and Staphylococcus aureus catheterrelated bacteremia. By day 80 after transplant, chronic graft-versus-host disease is documented by biopsies of the gastrointestinal tract and skin, and high-dose corticosteroids are begun. On day 90, she develops fever to 38.9 C (102.0 F), pleuritic chest pain, cough, and hemoptysis. A chest radiograph shows a right lower lobe pulmonary infiltrate. A highresolution CT scan of the chest is shown.

Vancomycin and cefepime are begun. Which of the following should be done next? A. Remove the central venous catheter B. Begin amphotericin B C. Begin fluconazole D. Begin azithromycin Infectious Disease Medicine:Question 43 Two years after undergoing mitral valve replacement, a 48-year-old man has a cerebrovascular accident. Except for fever, general physical examination is noncontributory. The cardiac examination is unchanged from previous findings. A transesophageal echocardiogram shows an oscillating mass on the mitral valve but no evidence of perivalvular extension or abscess. Six sets of blood cultures grow Enterococcus faecalis, which is resistant to penicillin and ampicillin but sensitive to vancomycin. The laboratory also reports the absence of high-level resistance to gentamicin and streptomycin. A decision is made to treat medically, and the patient is begun on vancomycin, 1 g intravenously every 12 hours, and gentamicin, 80mg intravenously every 8 hours (the patient weighs 76kg [167 Ib). After 3 days of therapy, the laboratory reports that his vancomycin peak level is 32 g/mL with a trough level of 9 g/mL, and his gentamicin peak level is 3.2 g/mL with a trough level of 0.8 g/mL. The technician notes that the vancomycin peak and trough levels and the gentamicin trough level are in the desirable range but that the laboratorys therapeutic peak range for gentamicin is 4 to 8 g/mL. Repeat blood cultures show no growth, and complete

blood count and serum creatinine values are normal. Which of the following is most appropriate at this time? A. Increase the gentamicin dose; keep the vancomycin unchanged B. Decrease the interval between the gentamicin doses; keep the vancomycin unchanged C. Keep both the gentamicin and the vancomycin doses unchanged D. Increase the vancomycin dose; keep the gentamicin unchanged

Infectious Disease Medicine:Question 44 A 35-year-old woman who underwent cadaveric renal transplantation 12 months ago is hospitalized because of fever, headache, ataxia, and confusion. Three weeks ago, she attended a family reunion where she ate processed meats and cole slaw. One day after the reunion, she developed loose stools and low-grade fever that lasted for 24 hours. Yesterday (the day prior to hospital admission), fever returned and she developed a headache. Current medications are prednisone and azathioprine. She is allergic to penicillin, as she developed anaphylactic shock after receiving this drug. On physical examination, temperature is 39.4 C (103.0 F), pulse rate is 100/ min, respiration rate is 30/mm, and blood pressure is 90/60 mm Hg. On neurologic examination, she is confused and oriented only to person. Her neck is supple. Plantar responses are extensor bilaterally. The leukocyte count is 18,500/L. (with 20% band forms). Lumbar puncture is performed. Cerebrospinal fluid leukocyte count is 1500/L (with 50% neutrophils and 50% lymphocytes), glucose is 30 mg/dL (simultaneous plasma glucose is 81 mg/dL), and protein is 300 mg/dL. Gram stain of cerebrospinal fluid shows gram-positive bacilli. Which of the following antimicrobial agents should be started? A. Erythromycin B. Chloramphenicol C. Trimethoprim/sulfamethoxazole D. Vancomycin E. Levofloxacin

Infectious Disease Medicine:Question 45 A 67-year-old man is planning a 3-week fishing trip to the Amazon basin areas of Peru and Brazil. He has hypertension, diet-controlled diabetes mellitus, and no known allergies. Current medications are hydrochlorothiazide and lisinopril. Which of the following is the single most important vaccine for this traveler? A. Japanese encephalitis B. Yellow fever C. Cholera D. Haemophilus influenzae type b E. Meningococcal

Infectious Disease Medicine:Question 46 A 46-year-old man with diabetes mellitus develops a plantar foot ulcer. The ulcer can be probed to the first metatarsal head, indicative of osteomyelitis. Culture of the bone grows

imipenem-sensitive Pseudomonas aeruginosa, imipenem-sensitive Acinetobacter species, vancomycin-resistant and penicillin-resistant Enterococcus faecium, and methicillinresistant Staphylococcus aureus. Which of the following antibiotic agents is most appropriate for this patient? A. Ertapenem B. Ertapenem plus quinupristin/dalfopristin C. Imipenem D. Imipenem plus linezolid

Infectious Disease Medicine:Question 47 A 42-year-old man with known late-stage HIV infection is hospitalized because of a cough, fever of 5 weeks duration, night sweats, anorexia, and weight loss. He had been doing well until the current symptoms developed. HIV infection was diagnosed several years ago. His CD4 cell count had been as low as 10/L at a time when he had Pneumocystis carinii pneumonia. Several different antiretroviral regimens that included agents from all three classes were ineffective, but reasonable control was eventually achieved using a regimen of lamivudine, stavudine, indinavir, and ritonavir. His last plasma HIV RNA viral load was 310 copies/mL with a CD4 cell count of 185/L. An admission chest radiograph shows right hilar lymphadenopathy, a right lower lobe pulmonary infiltrate, and a right pleural effusion. The patient is placed in respiratory isolation. Two of two sputum specimens stain positive for acid-fast bacilli. Which of the following is the most appropriate treatment at this time? A. Clarithromycin and ethambutol B. Clarithromycin, ethambutol, and rifabutin C. Isoniazid, rifampin, and pyrazinamide D. Isoniazid, rifabutin, pyrazinamide, and ethambutol E. Await culture results before beginning antimycobacterial therapy Infectious Disease Medicine:Question 48 A 20-year-old male college student is evaluated because of a 4-day history of vesicular lesions on his back, chest, arms, face, and the inside of his cheeks. He also had mild fever, which has since resolved. He does not have respiratory or neurologic symptoms. Physical examination reveals several hundred lesions distributed in the aforementioned areas that are in various stages of development, including papules, vesicles, and small pustules. Some lesions are crusted. He does not have lymphadenopathy, his lungs are clear, and neurologic examination is normal. You diagnose chickenpox (primary varicella) and advise him to stay home until all lesions are crusted. You also advise him and family members to contact you if respiratory or neurologic symptoms develop or if the skin lesions do not resolve within 7 to 10 days. Which of the following is also appropriate at this time? A. Oral acyclovir B. Topical acyclovir in sufficient quantities to cover all lesions C. Home administration of intravenous acyclovir D. Oral antihistamines to be taken as needed for itching E. Topical antibacterial ointment to be applied to the pustular lesions Infectious Disease Medicine:Question 49 A 57-year-old male Russian immigrant is hospitalized because of cough and fever. He has limited English language skills and cannot provide a detailed personal or family medical history. A chest radiograph shows a right upper lobe pulmonary infiltrate. Cefotaxime and azithromycin are started, and the patient is placed in a private room. On the third hospital day, family members visit and state that the patient has several

relatives who had been incarcerated in Russia and have hemoptysis, fever, and cough. A sputum smear is obtained from the patient, which is strongly positive for acid-fast bacilli. Which of the following is the most appropriate management for health care workers who participated in this patients care? A. Tuberculin skin testing now and in 12 weeks B. Chest radiograph now; no further intervention if the chest film is normal C. Immediate initiation of isoniazid D. Immediate initiation of isoniazid, rifampin, ethambutol, and pyrazinamide E. Observation for clinical disease

Infectious Disease Medicine:Question 50 A 19-year-old man with multiple episodes of sinusitis and pneumonia is found to have abnormal serum lgG, IgA, and 1gM levels (all were more than 2 standard deviations below the normal mean). He had a month-long febrile illness several months ago that remitted spontaneously and remains undiagnosed. His older brother died of lymphoma at age 12 years, and a maternal uncle died during adolescence after his bone marrow stopped making blood cells. Which of the following viruses would best explain the cause of these illnesses in the three family members? A. HIV B. Cytomegalovirus C. Human T-cell lymphotropic virus-1 (HTLV-1) D. Human herpesvirus 6 E. Epstein-Barr virus Infectious Disease Medicine:Question 51 A 45-year-old man from Cape Cod, Massachusetts, developed a slowing expanding erythematous lesion on his back. His physician declined to obtain serologic studies for Lyme disease but started treatment with doxycycline for 21 days, and the lesion quickly improved. Two weeks later, the patient develops fevers, drenching night sweats, myalgias, and malaise. Other than a splenectomy 15 years ago following a motor vehicle accident, he is in good health and takes no medications. On physical examination, temperature is 40.0 C (104.0 F). Pallor and pale conjunctivae are present. There is no lymphadenopathy. Heart rate is regular. A grade 2/6 systolic murmur is heard. The lungs are clear. The abdomen has a long midline surgical scar. Bowel sounds are normal. The liver is palpable 4 cm below the right costal margin in the midclavicular line. There is no palpable spleen. Laboratory studies: Hemoglobin 7.1 g/dL Hematocrit2l % Leukocyte count 4000/L Platelet count 50,000/L Serum haptoglobin 1 mg/dL Serum aspartate aminotransferase 250 U/L Serum alanine aminotransferase 150 U/L Serum bilirubin 3.0 mg/dL Serum alkaline phosphatase 375 U/L Serum lactate dehydrogenase 575 U/L Which of the following should be done next to establish the diagnosis? A. Serologic studies for Lyme disease B. Ehrlichia antibody titer C. Serologic studies for Rocky Mountain spotted fever

D. Giemsa- or Wright-stained peripheral blood smear Infectious Disease Medicine:Question 52 A 40-year-old female hospital employee is referred for evaluation of a positive tuberculin skin test (10-mm induration). The patient was born in India and moved to the United States 10 years ago. She received bacille Calmette-Gurin vaccination as a child and remembers having at least two tuberculin skin tests in her teens and twenties, which she was told were positive. She has not been tested since that time and has never received antituberculous medications. The patient is symptom-free, has no chronic medical problems, and takes no medications. She knows of no definite exposure to anyone with active tuberculosis. A recent chest radiograph was normal. Which of the following is most appropriate for this patient? A. No chemoprophylaxis is necessary, since her positive tuberculin skin test reaction is most likely due to the bacille Calmette-Gurin vaccination B. No chemoprophylaxis is necessary, since she is a long-term tuberculin skin test reactor who is older than 35 years of age C. Administration of isoniazid, 300 mg daily for 9 months D. Administration of rifampin, 600 mg daily for 4 months Infectious Disease Medicine:Question 53 A 58-year-old man with a history of chronic alcoholism was admitted to the intensive care unit 3 days ago because of Streptococcus pneumoniae pneumonia and bacteremia. When admitted, he was hypothermic, hypotensive (systolic blood pressure of 80 mm Hg), tachypneic, and hypoxemic. He also had lactic acidosis and soon after admission developed oliguria. The patient is currently receiving appropriate antibiotic therapy, saline volume repletion, vasopressor agents, ventilatory support, sedation, stress ulcer prophylaxis, and low-molecular-weight heparin. He is hemodynamically stable and has adequate oxygenation. Which of the following additional therapeutic measures is currently accepted as potentially beneficial for this patient? A. Administration of a corticosteroid B. Nutritional support C. Administration of N-acetylcysteine D. Administration of nitric oxide E. Administration of colloid Infectious Disease Medicine:Question 54 A 76-year-old woman with diabetes mellitus and end-stage renal disease, who requires chronic hemodialysis via a left subclavian vein catheter, develops chills and fever while in the dialysis center. The patient is hospitalized. Two sets of blood cultures are drawn on admission, intravenous vancomycin is begun, and the fever resolves. On the third hospital day, the patient again becomes febrile, and two additional sets of blood cultures are drawn. Also on the third day, the microbiology laboratory reports that three of the four original blood culture bottles are growing gram-positive cocci in chains. On the fourth hospital day, the second two sets of blood cultures are also reported to be growing gram-positive cocci in chains. In addition, subculture of the original two sets grows nonhemolytic colonies on blood agar plates (presumptively identified by the laboratory as an enterococcus). On the fourth hospital day, the subclavian vein catheter is removed, and the catheter tip is cultured semiquantitatively. Which of the following is the most appropriate antibiotic therapy pending definitive identification and sensitivity testing of the causative pathogen? A. Continue intravenous vancomycin B. Change to clindamycin. C. Change to quinupristin/dalfopristin D. Change to moxifloxacin

E. Change to linezolid Infectious Disease Medicine:Question 55 A 34-year-old woman has newly diagnosed HIV infection. Although she has fatigue and mild anorexia, her weight has been stable. Her CD4 cell count is 230/L with a plasma HIV RNA viral load of 99,000 copies/mL. Zidovudine, lamivudine, and abacavir are begun. One week later, she develops a diffuse rash, nausea, nonproductive cough and fever to 38.9C (102.0 F). Which of the following is most appropriate at this time? A. Substitute another recommended agent for abacavir C. Substitute another recommended agent for zidovudine D. Continue all medications; obtain a chest radiograph for possible Pneumocystis carinii pneumonia E. Continue all medications; begin treatment of symptoms Infectious Disease Medicine:Question 56 A 29-year-old man who is undergoing chemotherapy for acute myeloid leukemia develops blisters on his soft palate that quickly ulcerate. His only other symptoms are low-grade fever and weakness. Broad-spectrum antibiotics were started yesterday because of febrile neutropenia. On physical examination, temperature is 38.1 C (100.6 F). Several shallow ulcers without exudate or odor are present on his palate and pharynx. He is able to swallow without much difficulty. There are no skin lesions, and his lungs are clear. Serum creatinine is 1.0 mg/dL, and a chest radiograph is normal. A Tzanck smear of an oral lesion, which was obtained by gently scraping the lesion with a tongue blade, reveals multinucleated giant cells. Which of the following is the most appropriate therapy for this patients oral ulcers? A. Mupirocin ointment applied 3 times daily for 10 days B. Acyclovir, 400 mg orally 5 times daily for 10 days C. Ganciclovir, 5mg/kg intravenously every 12 hours for 14 days D. Codeine, 30 to 60 mg orally every 4 to 6 hours as needed E. Methylprednisolone, 30 to 60 mg intravenously every 6 hours for 3 days

Infectious Disease Medicine:Question 57 A 19-year-old female college student comes to the student health service because of dysuria and urinary urgency. A urinary tract infection is diagnosed, and the patient is given a 3-day course of trimethoprim/sulfamethoxazole. She forgot to mention that she had taken the same antibiotic for a urinary tract infection 1 month before school started. Three days later, the patient reports that the dysuria improved during the first 2 days of therapy but became worse on day 3. Which of the following most likely explains this patients failure to improve? A. She is infected with a trimethoprim/sulfamethoxazole-resistant organism B. She has been noncompliant with treatment C. She does not have a urinary tract infection D. She has a urinary tract reinfection

Infectious Disease Medicine:Question 58 After performing an arterial puncture, a physician sustains a needlestick injury from a patient whose HIV status is not known. On questioning, the patient admits to a history of injection drug use and multiple sexual partners but states that he has been drug free and abstinent for 1 year. He has never been tested for HIV and consents to being tested on the day of the injury. Which of the following is the most appropriate management for the physician? A. Begin three-drug antiretroviral therapy immediately; continue therapy for 1 month

B. Await results of the patients HIV antibody test before beginning antiretroviral therapy, as the results will be available in 24 hours C. Begin three-drug antiretroviral therapy immediately; discontinue therapy immediately if the patients HIV antibody test is negative D. Obtain HIV RNA viral load testing by polymerase chain reaction for both the patient and the physician now and in 6 weeks E. Obtain baseline serologic testing for HIV, hepatitis B, and hepatitis C for both the patient and the physician; repeat testing in 6 weeks and again in 6 months

Infectious Disease Medicine:Question 59 A 57-year-old male city government clerk has missed 3 days of work because of increasingly severe malaise, fever, dry cough, shortness of breath, and headache. On the fourth day of his illness, he is brought to the emergency department by ambulance. On physical examination, temperature is 39.0 C (102.2 F), and respiration rate is 32/mm. Examination of the chest reveals only a few scattered crackles bilaterally. The remainder of the examination is unremarkable. Hemoglobin 14.2 g/dL Leukocyte count 1 5,300/L Platelet count 1 90,000/L Blood urea nitrogen 61 mg/dL Serum creatinine2.0 mg/dL A chest radiograph shows vague scattered pulmonary infiltrates and a widened mediastinum. A sputum specimen cannot be obtained. Lumbar puncture is performed. Cerebrospinal fluid leukocyte count is 1050/L, erythrocyte count is 20/L, glucose is 44 mg/dL (simultaneous plasma glucose is 155 mgldL), and protein is 190 mg/dL. A Gramstained cerebrospinal fluid specimen shows multiple gram-positive bacilli. Which of the following is the most appropriate initial therapy for this patient? A. Ceftriaxone, ampicillin, and vancomycin B. Ciprofloxacin or doxycycline plus clindamycin and vancomycin C. Ceftriaxone, penicillin G, azithromycin, and rifampin D. Ciprofloxacin and doxycycline Infectious Disease Medicine:Question 60 A 25-year-old man comes to the emergency room because of severe pain and swelling of the right arm that began at the site of a small cut. He also reports fever, generalized malaise, and fatigue. On physical examination, the patient appears acutely ill. Temperature is 40.0 C (104.0 F), pulse rate is 135/ min, respiration rate is 35/mm, and blood pressure is 80/40 mm Hg. The right arm is erythematous with tense edema and several bullous lesions. Hemoglobin 9.0 g/dL Hematocrit 27% Leukocyte count27,350/L Blood urea nitrogen 45 mg/dL Serum creatinine 1.5 mg/dL Serum electrolytes: Sodium 133 meq/L Potassium4.8 meq/L Chloride 102 meq/L Bicarbonate 15 meq/L Results of blood cultures and a surgical consultation are pending. Which of the following is the most appropriate diagnostic imaging study at this time?

A. Plain radiograph of the arm B. CT scan of the arm C. MRI scan of the arm D. Ultrasound examination of the arm Infectious Disease Medicine:Question 61 A 53-year-old man underwent open reduction and internal fixation of a fractured tibia. The patient has diabetes mellitus and end-stage renal disease and requires hemodialysis by means of an arteriovenous graft in the left upper extremity. Three weeks postoperatively, his surgical incision became inflamed, with an open section and drainage of cloudy yellow fluid. Culture of the discharge grew methicillin-resistant Staphylococcus aureus (MRSA) that was resistant to erythromycin, clindamycin, and tetracycline but sensitive to vancomycin and trimethoprim/sulfamethoxazole. The patient was treated intermittently with vancomycin, 500 mg intravenously. Two months later, the surgical incision is unchanged. Culture of the discharge now grows Enterococcus faecalis in addition to MRSA. Both pathogens are resistant to vancomycin. Polymerase chain reaction shows that the MRSA is vanA ligase-positive. Which of the following is the most appropriate antibiotic agent for this patient? A. Linezolid B. Trimethoprim/sulfamethoxazole C. Clindamycin D. Imipenem E. Quinupristin/dalfopristin

Infectious Disease Medicine:Question 62 A 20-year-old female college student develops painful lesions on her vulva associated with fever and myalgias. She has no history of similar lesions but did have unprotected sexual intercourse with a new partner 1 week ago. On physical examination, temperature is 37.9 C (100.2 F). Several genital ulcers and vesicles are present. The remainder of the examination is normal. Which of the following is the most appropriate treatment for this patients lesions? A. Oral acyclovir or valacyclovir for 7 to 10 days B. Oral valganciclovir for 7 to 10 days C. Oral zidovudine for 7 to 10 days D. Oral prednisone for 7 to 10 days E. No treatment is indicated

Infectious Disease Medicine:Question 63 A 32-year-old man with advanced AIDS is hospitalized because of refractory diarrhea. Colonoscopy shows diffuse colitis, and biopsy findings are consistent with cytomegalovirus colitis. Which of the following isolation precautions is most appropriate for this patient? A. Contact isolation B. Droplet isolation C. Airborne isolation (negative pressure) D. Contact and airborne isolation E. Standard precautions; no special isolation precautions are needed

Infectious Disease Medicine:Question 64 A 35-year-old woman had been vacationing in Cancun, Mexico, for the past week. This morning, she noted the sudden onset of crampy, watery diarrhea but has not passed any blood or mucus with her stools. She has no fever, and her only other symptoms are mild

nausea and anorexia of 1 days duration. She has a history of occasional migraine headaches, has no known allergies, and takes no prescription medications. Which of the following organisms is most likely to be causing this patients diarrhea? A. Enterotoxigenic Escherichia coli (ETEC) B. Clostridium difficile C. Entamoeba histolytica D. Strongyloides stercoralis E. Salmonella enteritidis

Infectious Disease Medicine:Question 65 A 38-year-old man with HIV infection, who has recently moved to the area, is found to have elevated liver enzyme values. HIV positivity was diagnosed approximately 5 years ago. The patient is an injection drug user who has been in and out of jail and has not received consistent antiretroviral therapy. He currently takes no medications and feels well. His last CD4 cell count, approximately 6 months ago, was 554/L. Physical examination is unremarkable except for a barely palpable spleen tip. There is no hepatomegaly and no findings suggestive of cirrhosis. Serum alanine aminotransferase is 172 U/L, and serum aspartate aminotransferase is 129 U/L. Serum bilirubin and alkaline phosphatase values are normal. Hepatitis testing shows the following: Antibodies to hepatitis C virus (anti-HCV)Negative Antibodies to hepatitis B surface antigen (anti-HB5Ag)Positive Antibodies to hepatitis B core antigen (anti-HBcAg)Positive Hepatitis B surface antigen (HB5Ag)Negative Antibodies to hepatitis A virus (anti-HAV)Positive Which of the following is the most appropriate next step in evaluating this patients liver enzyme abnormalities? A. No further testing is required B. Qualitative polymerase chain reaction for HCV RNA C. Percutaneous liver biopsy D. Hepatitis G virus serologic studies E. Blood culture for mycobacteria Infectious Disease Medicine:Question 66 An 89-year-old woman is hospitalized because of mental confusion and fever. The patient lives at home. She has had no serious illnesses and takes no long-term medications. On physical examination, temperature is 39.4 C (103.0 F). Examination of the lungs discloses crackles over the posterior right lower lobe. A chest radiograph confirms an infiltrate in this area. Community-acquired pneumonia is diagnosed, and an intravenous fluoroquinolone is begun. The patient initially becomes afebrile and more alert. However, on the third hospital night, she becomes hypotensive and develops a rapid heart rate. An electrocardiogram shows ventricular tachycardia. After resuscitation and restoration of normal sinus rhythm by cardioversion, which of the following antibiotic regimens would be most appropriate? A. Continue the fluoroquinolone B. Change to ceftriaxone C. Change to ceftriaxone plus a macrolide D. Change to ceftriaxone plus gentamicin E. Change to ampicillin/sulbactam

Infectious Disease Medicine:Question 67 A 32-year-old pregnant woman who works in a daycare center develops lesions on her back, chest, and arms. She also has fever, an increasing cough, and shortness of breath. There have been two cases of chickenpox at the daycare center. The patient has never had this infection. On physical examination, she is obviously dyspneic while talking and has skin lesions in various stages of development, including papules, vesicles, and pustules. A chest radiograph shows bilateral diffuse interstitial and nodular infiltrates. The patient is hospitalized and placed in airborne isolation. Which of the following antiviral agents is most appropriate at this time? A. Oseltamivir orally B. Ganciclovir orally C. Ganciclovir intravenously D. Acyclovir orally E. Acyclovir intravenously Infectious Disease Medicine:Question 68 A 61-year-old man who is receiving consolidation chemotherapy for acute myeloid leukemia develops a fever without chills or other symptoms. His last measured leukocyte count was 950/L with an absolute neutrophil count of 390/L. Platelet count was 145,000/L. He lives with his wife, and their home is within 5 miles of his physicians office and the hospital. On physical examination in his physicians office, temperature is 39.0 C (102.2 F). Other vital signs are normal, and there is no rash. A Hickman catheter is in place and shows no evidence of infection. The remainder of the examination is normal. Which of the following antimicrobial agents is most appropriate for treating this patient initially? A. Oral trimethoprim/sulfamethoxazole B. Oral ciprofloxacin plus amoxicillin/clavulanate C. Intravenous vancomycin D. Intravenous cefepime E. Intravenous vancomycin plus cefepime Infectious Disease Medicine:Question 69 Five patients with septic shock are being treated in an intensive care unit. In which one of the five patients would adjunctive use of recombinant human activated protein C (aPC or drotrecogin alfa [activated]) be most useful and least likely to cause additional complications? A. A 74-year-old man with an ischemic bowel B. A 68-year-old woman with staphylococcal bacteremia 1 day after undergoing coronary artery bypass graft surgery C. A 34-year-old male stem-cell-transplant recipient with pancytopenia and candidemia D. A 50-year-old woman with a ruptured appendix who is scheduled to undergo surgery in 2 hours E. A 55-year-old man with hepatic and renal dysfunction and blood cultures that are positive for Pseudomonas Infectious Disease Medicine:Question 70 A 24-year-old female medical assistant wants to be tested for HIV because of concerns about possible work-related exposure to bloodborne pathogens, although she denies any specific invasive incident. She has been in a monogamous heterosexual relationship for the past 10 months. She and her partner have occasionally engaged in unprotected sex. The patient is anxious about her HIV status but otherwise feels well, is asymptomatic, and has not had any recent illnesses. Routine serologic testing by enzyme immunoassay is positive. Western blot shows a

single band corresponding to the p24 protein, which is interpreted as indeterminate. A plasma HIV RNA viral load is 375 copies/mL. Which of the following best describes the test results and appropriate management for this patient? A. This is an indeterminate result; repeat the serologic studies at 6 weeks, 3 months, and 6 months B. This is an indeterminate result; obtain a CD4 cell count, as a count of less than 350/L indicates likely HIV infection C. This is a false-positive test result; there is no need for further diagnostic testing D. This is a false-positive test result; recheck the plasma HIV RNA viral load in 3 months E. The patient has HIV infection; there is no need for further diagnostic testing

Infectious Disease Medicine:Question 71 A 64-year-old male lawyer has a 2-day history of a painful rash in a bandlike pattern on one side of his chest. The rash extends to his back but does not cross the midline. The patient has hypertension and diabetes mellitus for which he takes two antihypertensive medications and an oral hypoglycemic agent. He had chickenpox at 6 years of age. There is no history of immunodeficiency. Physical examination reveals several groups of vesicular lesions in the distribution of the right T5 dermatome. He does not have lymphadenopathy, his lungs are clear, and neurologic examination is normal. Which of the following is the most appropriate treatment at this time? A. Oral acyclovir, valacyclovir, or famciclovir for 7 days B. Topical acyclovir or penciclovir in sufficient quantities to cover all lesions C. A topical antibacterial ointment in sufficient quantities to cover all lesions D. Home administration of intravenous acyclovir E. Oral prednisone; no antiviral agents are indicated Infectious Disease Medicine:Question 72 A 35-year-old woman comes to your office for the first time. A cardiologist in another city told her that she has mitral valve prolapse and therefore needs to take antibiotics prior to dental procedures. A copy of her echocardiogram report states that she has Dopplerdemonstrated mitral regurgitation. She will have a tooth extracted in 4 days and asks for a 2-day supply of clindamycin, which she was given previously. She states that she is allergic to penicillin and that she had to go to an emergency department after she became flushed and her throat closed-up the last time she took this antibiotic. Physical examination is normal except for a soft murmur of mitral regurgitation. According to current guidelines from the American Heart Association, which of the following is most appropriate for prophylaxis prior to this patients dental procedure? A. Tell her that she does not need prophylaxis for this procedure B. Prescribe a 2-day course of cephalexin to start 30 minutes before the procedure C. Prescribe a 7-day course of cephalexin to start today D. Prescribe a 2-day course of clindamycin to start 30 minutes before the procedure E. Prescribe a single 600-mg dose of clindamycin to be taken 1 hour before the procedure

Infectious Disease Medicine:Question 73 A 47-year-old man recently began chemotherapy for acute myeloid leukemia. His leukocyte count subsequently fell below 500/L, and he was monitored closely for infection. When he developed a fever, ceftazidime and vancomycin were begun, following which his temperature decreased.

A bone marrow examination 2 weeks after the conclusion of chemotherapy showed persistent tumor, and another course of chemotherapy was begun. On the 25th day of hospitalization, at a time when he had no measurable circulating granulocytes, he developed another fever. This time, his chest radiograph showed a small abnormal patch in the right mid-lung area. Sputum production was not increased. Imipenem was substituted for ceftazidime and vancomycin, but his temperature did not decrease, and the abnormal area on the chest radiograph grew larger. He refused bronchoscopy. His fever continues and he has developed chills, which he tolerates poorly. His serum creatinine level has increased from 1.1 mg/dL at baseline to 1.9 mg/dL. The dose of imipenem is decreased. Which of the following antimicrobial agents should be started next? A. Fluconazole B. Flucytosine C. Amphotericin B colloidal dispersion D. Liposomal amphotericin B E. Nystatin Infectious Disease Medicine:Question 74 A 37-year-old man has HIV infection that was diagnosed 3 years ago when he developed bilateral cytomegalovirus retinitis. The retinitis was treated with intravenous ganciclovir and intraocular ganciclovir implants and has remained in remission. Antiretroviral therapy with zidovudine, lamivudine, and nelfinavir was also begun at that time. Since beginning therapy, his CD4 cell count has increased from a low of 1 24iL to a high of approximately 450/L over the last 18 months. His plasma HIV RNA viral load has been consistently undetectable. The patient has no other AIDS-related complications and feels well. His last ophthalmologic examination showed no signs of active cytomegalovirus retinitis. In addition to his antiretroviral medications, he takes valganciclovir (900 mg orally daily), trimethoprim/sulfamethoxazole (1 double-strength tablet daily), and azithromycin (1200 mg orally weekly). If possible, the patient would like to reduce the number of medications that he is taking. Which of the following medications can be safely discontinued at this time? A. Valganciclovir B. Valganciclovir if a cytomegalovirus IgM titer is negative C. Nelfinavir D. Trimethoprim/sulfamethoxazole and azithromycin E. Valganciclovir, trimethoprim/sulfamethoxazole, and azithromycin Infectious Disease Medicine:Question 75 A 65-year-old woman, who was hospitalized because of severe abdominal pain, underwent surgery for a bowel infarction caused by a volvulus. Postoperatively, she required mechanical ventilation and had a difficult initial course in the intensive care unit. After intravenous broad-spectrum antibiotics (imipenem and vancomycin) were administered, she did fairly well for about 1 week. On the 12th postoperative day, tube feedings are started. The patient subsequently develops diarrhea but is afebrile and has diminishing abdominal pain and distention. Stool specimens are negative for leukocytes, and an assay forClostridium difficile toxin is negative on two occasions. Stool cultures grow no enteric pathogens but do show a heavy growth of Enterococcus faecium that was resistant to ampicillin and vancomycin. Which of the following is most appropriate for managing this patient at this time? A. Contact isolation B. Standard precautions; no special isolation precautions are needed C. Add oral vancomycin to her current regimen D. Change imipenem to ceftazidime and metronidazole E. Check for an intestinal leak

Infectious Disease Medicine:Question 76 A 55-year-old man underwent nonmyeloablative stem cell transplantation as part of the treatment regimen for chronic myelogenous leukemia. Before signs of engraftment occurred, he developed hemorrhagic cystitis followed by bilateral pneumonia. Urine and bronchoalveolar lavage fluid cultures did not grow bacteria or fungi. The pneumonia progressed relentlessly despite aggressive antibiotic and antifungal therapy, and the patient died 1 week after the onset of the pulmonary infection. Which of the following viruses was the most likely cause of this patients terminal illness? A. An adenovirus B. Cytomegalovirus C. Epstein-Barr virus D. Respiratory syncytial virus E. Influenza virus

Infectious Disease Medicine:Question 77 A previously healthy 25-year-old woman comes to the emergency department because of a 2-day history of fever, chills, shortness of breath, left-sided pleuritic chest pain, and dizziness. She has a nonproductive cough and is barely able to speak because of increasing dyspnea. The patient also has been vomiting all day but does not have diarrhea or abdominal pain. On physical examination, temperature is 39.0 C (102.2 F), pulse rate is 110/ min, respiration rate is 44/mm, and blood pressure is 92/60 mm Hg. Crackles are heard in the left lower lung field. Arterial blood gas studies (with the patient breathing room air) show PCO2 of 26 mm Hg and PO2 of 85 mm Hg. A chest radiograph shows a dense infiltrate in the left lower lung field. The patient is given one dose each of ceftriaxone and azithromycin. Because of increasing respiratory failure, she is admitted to the intensive care unit for intubation and mechanical ventilation. Over the next 2 hours, she receives 5 liters of crystalloidcontaining fluids. Temperature now is 39.5 C (103.1 F), pulse rate is 100/ min, and blood pressure is 84/56 mm Hg. Which of the following is the most likely explanation for this patients continued hypotension? A. Ineffective antibiotic therapy B. Low systemic vascular resistance C. High cardiac index D. Poor oxygenation E. Elevated pulmonary capillary wedge pressure Infectious Disease Medicine:Question 78 A 48-year-old man is hospitalized because of cough, hemoptysis, fever, weight loss, and dyspnea. He has not felt well for at least 3 months because of the progressive cough and production of bloody sputum. The patient is homeless and has lived in a local mission periodically. Medical history is noncontributory, although he has had only limited medical care. On physical examination on admission, he appears cachectic. Temperature is 39.2 C (102.6 F), pulse rate is 110/ min, and respiration rate is 22/mm. Significant findings include poor dentition and scattered expiratory wheezes. The leukocyte count is 15,500/L with a left shift. A chest radiograph shows a right upper lobe infiltrate with a suggestion of cavity formation. The patient is placed in respiratory isolation, and antibacterial therapy for communityacquired pneumonia and aspiration is begun. On the second hospital day, admission blood cultures show no growth; a routine sputum culture is growing normal respiratory

flora. One of two sputum smears is positive for acid-fast bacilli. Information from the local health department indicates no significant drug resistance among tuberculosis isolates in the area. Antibiotics are discontinued. Which of the following is the most appropriate treatment for this patient? A. Start daily isoniazid and rifam pin now; plan for a 36-week course B. Start isoniazid, rifampin, and ethambutol three times weekly by directly observed therapy (DOT) now; plan for a 24-week course C. Start daily isoniazid, rifampin, pyrazinamide, streptomycin, and ethambutol now for 8 weeks; then reduce administration to three times weekly by DOT for 16 weeks D. Start daily isoniazid, rifampin, pyrazinamide, and ethambutol now for 2 weeks, then reduce to twice weekly by DOT for 6 weeks; then change the regimen to isoniazid and rifam pin twice weekly by DOT for 16 weeks E. Defer treatment until the species of acid-fast bacilli has been identified

Infectious Disease Medicine:Question 79 A 29-year-old woman is seen for ongoing management of HIV infection that was diagnosed 9 months ago. Her initial CD4 cell count was 194/L, and her plasma HIV RNA viral load was 56,780 copies/mL. She was immediately started on triple-drug highly active antiretroviral therapy (HAART) consisting of zidovudine (300 mg twice daily), didanosine (400 mg at bedtime), and nevirapine (200 mg twice daily). Trimethoprim/sulfamethoxazole was also begun. Her response to treatment is shown: Months Since Initiation of HAART CD4 Cell Count (L) Plasma HIV RNA Viral Load (copies/mL) 0 194 56,780 1 172 18,420 3 168 6810 6 185 5325 7 192 6220 The patient claims to have adhered her medication schedule and has missed only two doses of all medications since she started therapy. She looks and feels well. Which of the following is the most appropriate management at this time? A. Continue her current regimen; recheck her HIV RNA viral load in 3 months B. Order an HIV genotype; change her regimen on the basis of genotype results and treatment history C. Start three new antiretroviral medications now D. Recheck her CD4 cell count now; if greater than 200/L, continue her current regimen E. Assume nevirapine failure; change to efavirenz Infectious Disease Medicine:Question 80 A 52-year-old woman who has non-Hodgkins lymphoma and is receiving her third course of chemotherapy develops a fever. Her absolute neutrophil count has been less than 100/L for 2 weeks, but she had not had fever or other unexpected signs or symptoms until now On physical examination, temperature is 38.9 C (102.0 F). The remainder of the examination is noncontributory, the Hickman catheter appears normal, and blood cultures show no growth. Cefepime and gentamicin are begun. Three days later, the Hickman catheter tunnel shows signs of inflammation. Vancomycin is added to the regimen, and the catheter is removed. Culture of the catheter tip shows no growth. The patient remains severely neutropenic. Six days after starting the antibiotic regimen, she continues to have daily fevers to 39.0 C (102.2 F) and no subjective or objective

changes on physical examination. Which of the following is most appropriate at this time? A. Stop cefepime and gentamicin; add amphotericin B B. Continue all three antibiotics; add amphotericin B C. Continue all three antibiotics; add ciprofloxacin D. Stop all three antibiotics; start ciprofloxacin E. Make no changes to the antibiotic regimen Infectious Disease Medicine:Question 81 A 29-year-old male landscaper is evaluated by his primary care physician because of a 1day history of fever and increasing shortness of breath. On physical examination, temperature is 39.3 C (102.7 F), and respiration rate is 34/mm. Examination of the chest reveals profuse crackles bilaterally. A chest radiograph shows bilateral patchy pulmonary infiltrates involving at least four lobes. A Gram-stained sputum specimen shows large numbers of polymorphonuclear leukocytes with pink-staining nuclei and small, scattered, pleomorphic gram-negative rods. Blood cultures showed no growth, but cultures of tracheal aspirates grew Francisella tularensis. Which of the following is the optimal first-line drug for treating this patients infection? A. Chioramphenicol B. Ciprofloxacin C. Doxycycline D. Ceftriaxone E. Gentamicin

Infectious Disease Medicine:Question 82 A 39-year-old man with longstanding HIV infection and previously diagnosed AIDS develops blurred vision in his right eye. He is not receiving antiretroviral therapy, and his last known CD4 cell count was 4/L. Medical history is otherwise noncontributory. On physical examination, his sclerae and conjunctivae are normal, and his pupils are equal, round, and reactive to light and accommodation. Funduscopic examination shows several hemorrhages and exudates in the right eye. An ophthalmologist sees the patient that same day, confirms your findings, and reports that the lesions are most consistent with cytomegalovirus retinitis. Current laboratory studies include a leukocyte count of 4800/L (92% neutrophils), a normal platelet count, and a serum creatinine of 2.8 mg!dL. Which of the following is the most appropriate initial therapy for this patient? A. Ganciclovir by intravitreal injection into the right eye; no systemic therapy B. Three antiretroviral agents; no cytomegalovirus therapy C. Intravenous ganciclovir or oral valganciclovir plus evaluation for antiretroviral therapy D. Intravenous acyclovir plus evaluation for antiretroviral therapy E. High-dose oral prednisone plus evaluation for antiretroviral therapy

Infectious Disease Medicine:Question 83 A 34-year-old nurse reports a needlestick injury. After drawing blood from a patient, the nurse inadvertently stuck the needle into his own finger. The source patient is known to be positive for hepatitis B surface antigen. The nurse was vaccinated against hepatitis B when he was hired 3 years ago. He completed the series of three injections but has never had serologic confirmation of his response. Which of the following post-exposure options is most appropriate for this health care worker? A. Administer hepatitis B immune globulin immediately and restart his immunization sequence B. Check his antibody response to the hepatitis B vaccination; if antibodies are inadequate, administer hepatitis B immune globulin and restart his immunization

sequence C. Check his antibody response to the hepatitis B vaccination; if antibodies are adequate, administer only hepatitis B immune globulin D. As the nurse has completed his hepatitis B vaccination series, no intervention is necessary. Infectious Disease Medicine:Question 84 A 46-year-old man with diabetes mellitus had a recurrent plantar foot ulcer over the second left metatarsal head that did not probe to bone. Culture of deep soft tissue grew both Bacteroides fragilis and methicillin-resistantStaphylococcus aureus that was sensitive to vancomycin, linezolid, and quinupristin/dalfopristin but was resistant to all other antibiotics tested. Because of an allergy to vancomycin, the patient was started on linezolid, 600mg orally every 12 hours, plus metronidazole, 500 mg orally every 6 hours. He is re-evaluated 2 weeks later. The plantar ulcer is somewhat smaller, but he now has a petechial rash on his legs. Temperature is normal, and physical examination is otherwise unremarkable. Hemoglobin is 10.8 g/dL, the leukocyte count is 2500/L (with an absolute neutrophil count of 1 000/L), and the platelet count is 22,000/L. Which of the following is most likely causing this patients petechial rash? A. An adverse reaction to metronidazole B. An adverse reaction to linezolid C. Disseminated intravascular coagulation due to methicillin-resistant Staphylococcus aureus D. Hypersensitivity vasculitis due to antibiotic therapy E. Thrombotic thrombocytopenic purpura Infectious Disease Medicine:Question 85 A 30-year-old woman was hospitalized after the sudden onset of severe headache, stiff neck, and vomiting. Evaluation revealed a subarachnoid hemorrhage secondary to a leaking aneurysm. She was taken to the operating room, where the aneurysm was clipped and a ventriculostomy tube was placed to drain CSF.Four days postoperatively, she develops fever, worsening headache, and change in mental status. On physical examination, temperature is 39.4 C (103.0 F), pulse rate is 100/ min, respiration rate is 24/mm, and blood pressure is 120/70mm Hg. The patient is confused. The ventriculostomy tube is draining clear CSF. All surgical sites are clean without evidence of drainage. The leukocyte count is 15,000/L without a left shift. Urinalysis and chest radiograph are normal. Cerebrospinal fluid analysis shows a leukocyte count of 500/L (with 90% neutrophils), erythrocyte count of 900/L, glucose of 30 mg/dL (simultaneous plasma glucose of 85 mg/dL), and protein of 150 mg/dL. A Gram stain of CSF is negative. Which of the following empiric antimicrobial regimens should be initiated at this time? A. Vancomycin B. Vancomycin plus rifampin C. Vancomycin plus ceftriaxone D. Vancomycin plus ceftazidime E. No therapy is required, as this patient has postoperative chemical meningitis Infectious Disease Medicine:Question 86 A 22-year-old male college student comes to the emergency department in June because of fever, chills, myalgias, and a severe bilateral frontal headache. The patient recently went camping in North Carolina. On physical examination, temperature is 39.0 C (102.2 F), and blood pressure is 110/72 mm Hg. Findings include a petechial rash that is most prominent on the wrists and hands (including the palms), conjunctival injection, and mild meningismus. Neurologic examination is normal. Hemoglobin 10.0 g/dL

Hematocrit 30% Leukocyte count 9000/L Platelet count 80,000/L Blood urea nitrogen 35 mg/dL Serum creatinine 1.4 mg/dL Serum electrolytes: Sodium 128 meq/L Potassium4.0 meq/L Chloride 97 meq/L Bicarbonate2l meq/L Serum bilirubin 1 .8 mgldL Serum alkaline phosphatase 140 U/L Serum alanine aminotransferase 300 U/L Serum aspartate aminotransferase 250 U/L Which of the following organisms is most likely causing this patients symptoms? A. Rickettsia rickettsii B. Neisseria meningitidis C. Borrelia burgdorferi D. Babesia microti Infectious Disease Medicine:Question 87 A 34-year-old woman with schizophrenia was found to have leukemia after she was seen in a hematology clinic because of an elevated leukocyte count. She was referred from a psychiatric hospital where she had recently been admitted for severe auditory and visual hallucinations. Review of old records from another hospital showed that she had leukemia 4 years ago but was lost to follow-up. The patient was subsequently hospitalized, and chemotherapy was started for chronic myelogenous leukemia with blast crisis. During a period of neutropenia, she developed a fever and was treated with fluconazole, cefepime, and vancomycin. The fever failed to resolve after 5 days, and amphotericin B by infusion was added to the treatment regimen. The patient subsequently developed hives and hypotension, and the infusion was stopped. After 1 hour, the infusion was restarted very slowly, but the hives promptly returned. An antihistamine and a corticosteroid were begun immediately. Which of the following should also be started for management of a potential fungal infection in this patient? A. No treatment is indicated because the risks of giving amphotericin B outweigh the benefits B. Begin a lipid formulation of amphotericin B C. Begin caspofungin D. Begin voriconazole E. Increase the dose of fluconazole Infectious Disease Medicine:Question 88 A 62-year-old man with chronic atrial fibrillation, chronic obstructive pulmonary disease, and obesity was hospitalized because of shortness of breath. The patient takes warfarin. Spiral CT scan of the chest was ordered on admission to rule out a pulmonary embolism. Because the patient was anxious during the CT scanning, he was given 2 doses of a short-acting benzodiazepine. He stopped breathing, and a code was called in the radiology suite. The patient was intubated and immediately admitted to the intensive care unit for observation. Twelve hours later, he becomes febrile. On physical examination, temperature is 39.0 C (102.2 F), pulse rate is 80/ min, and blood pressure is 130/72 mm Hg without administration of vasopressor agents. Hemoglobin 16.0 g/dL Leukocyte count 8900/L (normal differential)

INR 3.9 Blood urea nitrogen Normal Serum creatinine Normal Serum electrolytes Normal A chest radiograph shows some diffuse changes suggestive of early acute respiratory distress syndrome. Blood cultures are obtained, and empiric antibiotics are started. Which of the following is also most appropriate for managing this patient at this time? A. Supportive care B. Vitamin K C. An inferior vena cava filter D. Amphotericin B E. Recombinant human activated protein C (aPC or drotrecogin alfa [activated]) Infectious Disease Medicine:Question 89 A 34-year-old man returned from a 10-day trip to Europe with dysuria and a mild urethral discharge. A rapid probe test indicated that he had gonorrhea but not Chlamydia infection. While traveling, he had sexual relations with commercial sex workers. On the 8th day of his trip, he noted the beginning of a discharge and took some ciprofloxacin tablets that he had from a previous trip to a developing country. However, he had only mild relief after taking the tablets for 3 days. The patients only other medical condition is dyspepsia for which he takes ranitidine. Which of the following is the most likely reason for his lack of response to ciprofloxacin? A. His course of treatment was too short B. The causative organism was resistant to ciprofloxacin C. He has concomitant syphilis D. Ranitidine prevented the absorption of ciprofloxacin Infectious Disease Medicine:Question 90 A 28-year-old man with newly diagnosed HIV infection is hospitalized because of worsening cough, dyspnea, and fever. He has also lost 7 kg (15 Ib) over the past 2 months. Cough and low-grade fever developed 1 week prior to admission, and he was given levofloxacin, 500 mg daily. Because of HIV risk behavior, HIV testing was also done prior to hospitalization, and a positive result was reported a few days later. Over the next 7 days until his hospitalization, his cough and fever worsened and he became increasingly dyspneic. On physical examination on admission, temperature is 39.6 C (103.3 F) and respiration rate is 28/mm. Arterial blood gas studies with the patient breathing room air show a pH of 7.48, PCO2 of 28 mm Hg, and PO2 of 62 mm Hg. An admission chest radiograph reveals diffuse bilateral interstitial infiltrates. The patient is given supplemental oxygen and treated empirically with trimethoprim/sulfamethoxazole (20 mg/kg/day intravenously divided into every-8-hour doses), and ceftriaxone, 1 g intravenously every 12 hours. An induced sputum specimen shows Pneumocystis carinii organisms by direct fluorescent antibody stain. By the third hospital day, the patient has become severely dyspneic and hypoxemic and requires endotracheal intubation and mechanical ventilation. Arterial blood PO2 is now 82 mm Hg with the patient receiving 60% fractional inspired oxygen (FiO2) and 7.5 mm Hg of positive end-expiratory pressure (PEEP). Which of the following is the most appropriate treatment at this time? A. Change from trimethoprim/sulfamethoxazole to pentamidine B. Continue current antibiotics; add pentamidine C. Continue current antibiotics; begin highly active antiretroviral therapy D. Continue current antibiotics; add prednisone, 40 mg twice daily E. Continue current antibiotics; add azithromycin

Infectious Disease Medicine:Question 91 A 30-year-old man presents with a 4-day history of fever, chills, and ill-defined abdominal discomfort 1 week after returning from a trip to mostly rural areas of Cambodia and Vietnam. He has no significant medical history, has no allergies, and took no prescription medications during his trip. Prior to travel, he received hepatitis A and typhoid vaccines but took no other preventive measures. On physical examination, his temperature is 39.2 C (102.6 F). There is no rash or lymphadenopathy. Cardiopulmonary and abdominal examinations are normal. Which of the following diseases is the most important to consider in the initial management of this patient? A. Hepatitis B B. Yellow fever C. Amebic liver abscess D. Plasmodium falciparum malaria

Infectious Disease Medicine:Question 92 A 36-year-old woman is admitted to the intensive care unit from the emergency department after ingesting an intentional overdose of a long-acting barbiturate. Endotracheal intubation was performed in the emergency department. Other than being drowsy and minimally arousable, the patient seems to be doing well. The gastric tube used to irrigate her stomach is still in place and is being used for feedings. Which of the following additional measures is appropriate to prevent complications in this patient? A. Prophylactic antibiotics as long as the endotracheal tube is in place B. Nasal placement of the endotracheal tube C. Semi-erect positioning in bed D. Changing the endotracheal tube every 2 to 3 days E. A cooling blanket to prevent a rise in temperature above 37.0 C (98.6 F) Infectious Disease Medicine:Question 93 An 18-year-old man is evaluated because of fever and weight loss of 3 weeks duration. He had several episodes of bacterial pneumonia during childhood and a perirectal abscess that was slow to heal when he was 13 years old. Medical history is otherwise noncontributory. The leukocyte count is 12,400/L, the erythrocyte sedimentation rate is 95 mm/h, and liver enzyme values are mildly elevated. CT scan of the abdomen shows a large hepatic abscess, which is subsequently drained percutaneously. Culture of the abscess fluid grows Staphylococcus aureus, and intravenous nafcillin is begun. The patients recovery is slow but complete. A nitroblue tetrazolium test is subsequently performed, which is abnormal. Which of the following medications is most appropriate to minimize the risk of further infections in this patient? A. No long-term antibiotic agents or other drugs are needed B. Dicloxacillin daily C. Trimethoprim/sulfamethoxazole daily plus interferon- D. Low-dose prednisone daily E. lnterleukin-2 Infectious Disease Medicine:Question 94 A 35-year-old woman calls because she has had a tingling sensation of her vulva for about 4 hours. She denies fever, myalgias, or headache. Your records show that she has a history of recurrent genital herpes infections, occurring about 2 to 3 times yearly, that were occasionally treated with oral acyclovir. She reminds you that most of her episodes

are preceded by a tingling sensation similar to what she is experiencing now. Which of the following is the most appropriate initial treatment for this patient? A. Topical acyclovir for 5 days B. Oral acyclovir daily to be taken indefinitely C. Oral acyclovir, valacyclovir, or famciclovir for 3 to 5 days D. Oral prednisone plus oral acyclovir for 7 days E. No treatment is indicated Infectious Disease Medicine:Question 95 A mother brings her 16-year-old daughter to a gynecologist because of a report from a family planning clinic that the daughter has a Chlamydia infection. Initially, the daughter went to the clinic to learn about birth control. As part of the clinics screening program for sexually transmitted diseases, testing was performed that indicated that the daughter had a Chlamydia infection. She has had two sexual partners in her life, and her partners usually use condoms. Her mother asks how she could develop this infection at such a young age especially since she did not have symptoms. Which of the following is the most appropriate response? A. Chlamydia is very common in young people who have had even a limited amount of sexual experience B. Most cases ofChlamydia occur in older adults (> 35 years of age), but rare cases occur in adolescents C. The test result was probably a false positive, since growingChlamydia is very difficult D. The daughter probably acquired Chlamydia nonsexually, from a family pet or an inanimate object Infectious Disease Medicine:Question 96 A 63-year-old woman with leukemia has been receiving chemotherapy on the inpatient oncology floor. On the fifth hospital day, she develops a vesicular rash on her left flank that is pruritic and somewhat painful. The patient states that the rash is similar to a prior outbreak of shingles that she had last year, although the current lesions involve a larger area. Which of the following isolation precautions is most appropriate for this patient? A. Contact isolation B. Droplet isolation C. Airborne isolation (negative pressure) D. Contact and airborne isolation E. Standard precautions; no special isolation precautions are needed Infectious Disease Medicine:Question 97 A 48-year-old man is evaluated because of severe dyspnea and profound weakness. HIV infection was diagnosed approximately 15 months ago. His initial CD4 cell count was 294/L, and his plasma HIV RNA viral load was 115,500 copies/mL. An antiretroviral regimen was started, consisting of didanosine (400 mg at bedtime), stavudine (40 mg twice daily), indinavir (800 mg twice daily), and ritonavir (100 mg twice daily). His HIV RNA viral load has been undetectable for the past 8 months, and his CD4 cell count has increased to 138/L. The patient became ill approximately 5 days ago, when he developed nausea, vomiting, abdominal pain, severe myalgias, and anorexia. Over the last 24 hours, he has become progressively more short of breath. His wife mentions that he drinks one or two cans of beer on most days and does not use illicit drugs. Previous tests for chronic hepatitis A and B were negative. On physical examination, the patient is dyspneic and appears to be acutely ill. Temperature is 37.0 C (98.6 F), pulse rate is 122/ min, respiration rate is 32/mm, and blood pressure is 112/72 mm Hg. Arterial oxygen saturation is 99% by pulse oximetry with the patient breathing room air. He has mild scleral icterus. The lungs are clear. Abdominal examination discloses mild hepatomegaly; the spleen is not enlarged.

Forty-eight hours after admission, his chest radiograph is normal, and blood and urine cultures are negative. However, his clinical findings are unchanged. Current laboratory studies are as follows: Leukocyte count7200/L (71% segmented neutrophils, 12% band forms, 9% monocytes, 5% lymphocytes, 3% eosinophils) Blood urea nitrogen 62 mg/dL Serum creatinine 1.4 mg/dL Serum electrolytes: Sodium 136 meq/L Potassium 3.8 meq/L Chloride 108 meq/L Bicarbonate 9 meq/L Serum alanine aminotransferase 579 U/L Serum aspartate aminotransferase 310 U/L Serum bilirubin3.9 mg/dL Serum lactate dehydrogenase 1255 U/L Serum creatine kinase 894 U/L Venous blood lactic acid 9 mg/dL Which of the following is the most likely diagnosis? A. Acute viral hepatitis B. Acute bacterial sepsis C. Alcoholic hepatitis D. Lactic acidosis/hepatic steatosis E. Hypovolemic shock

ANSWERS
Infectious Disease Medicine:Question 1 The correct answer is C Educational Objectives Understand how to diagnose and treat influenza during a community outbreak. Critique This patient likely has influenza based on the clinical presentation and the epidemiologic information. Empiric treatment is appropriate for someone who presents within 48 hours with a compatible illness if influenza has been documented in the community. However, empiric therapy for influenza alone would be inappropriate if the patient possibly had pneumonia, which would require additional evaluation for a bacterial source. Therapy would also be inappropriate if the epidemiologic data were not consistent with influenza (for example, if a patient presents during the summer and has not recently traveled outside the Northern Hemisphere). Any of the four drugs listed can be used to treat influenza A, although only the neuraminidase inhibitors (zanamivir and oseltamivir) would be appropriate if influenza B had been documented in the community. A chest radiograph and blood cultures are unnecessary for a patient without concomitant medical problems and no signs of lower respiratory tract infection. A nasopharyngeal culture might be useful for epidemiologic purposes. However, it would not be helpful for deciding whether to treat this patient, as results of virus cultures require several days but therapy should be started within the first 2 days of symptom development and is continued for only 5 days. Antibacterial agents are not indicated because this patient has no suggestion of pneumonia or bacterial sinusitis.

Infectious Disease Medicine:Question 2 The correct answer is D

Educational Objectives Recall the indications for initiation of antiretroviral therapy. Critique There are many controversies concerning the proper time to initiate antiretroviral therapy. Experts agree that treatment is indicated for patients with an AIDS-defining condition, low CD4 cell count (< 200/ L), or severe symptoms. The timing is more controversial for asymptomatic patients. Most experts suggest beginning therapy when the CD4 cell count is between 200 and 350/L, irrespective of the HIV RNA viral load. The key to decision making for this patient is the presence of thrush. Thrush indicates a poor prognosis for survival and an increased likelihood of opportunistic complications in AIDS patients. Therefore, even though this patients CD4 cell count is above 200/L, he should be offered antiretroviral therapy at this time with one of several recommended three-drug regimens. Prophylaxis for Pneumocystis carinii pneumonia should also be started. Repeating the CD4 cell count and plasma HIV RNA viral load is not needed. The fact that the patient has a low viral load at the beginning of treatment may influence the choice of agents but not the decision to use three versus two drugs.

Infectious Disease Medicine:Question 3 The correct answer is D Educational Objectives Determine the role of various antibiotic agents for treating severe vancomycinresistantEnterococcus faecalis infection. Critique This is a complicated case study. When ampicillin is not a choice (because of resistance or intolerance), linezolid is probably the best available antibiotic for vancomycin-resistant strains of Enterococcus faecalis. Resistance of E. faecalis to linezolid is very rare, although it is prudent to evaluate this possibility. Not treating the pathogen is appropriate for a patient with transient bacteremia but is inadvisable for a critically ill patient who continues to have positive blood cultures. Piperacillin/tazobactam is a reasonable choice in terms of the treatment spectrum, but the likelihood of adverse reactions in a patient with a recent penicillin allergic reaction is very high. Quinupristin/dalfopristin is a reasonable alternative for resistant Enterococcus faecium, but less than 10% of strains of E. faecalis are susceptible to this agent. Although ampicillin can be given safely to patients with IgE-mediated allergies after completion of an appropriate desensitization protocol, other types of allergic reactions do not abate following desensitization. Infectious Disease Medicine:Question 4 The correct answer is B Educational Objectives Recognize the need for airborne isolation in a hospitalized patient at high risk for tuberculosis. Critique This patient is at high risk forMycobacterium tuberculosis infection because of his immunocompromised state and recent incarceration. He should therefore requires empiric airborne isolation (negative pressure). Airborne isolation is needed becauseM.

tuberculosis is spread by droplet nuclei, which are aerosolized when patients with pulmonary infection speak, cough, or sneeze. In immunocompromised patients, tuberculosis may not have the classic isolated upper lobe pattern on chest radiographs. Patients with suspected M. tuberculosis infection do not require contact or droplet isolation. Infectious Disease Medicine:Question 5 The correct answer is C Educational Objectives Recall how to treat life-threatening increased intracranial pressure in a patient with HIV infection and cryptococcal meningitis. Critique This patient with untreated HIV infection has cryptococcal meningitis, which now occurs primarily in patients who do not benefit from or refuse to take antiretroviral therapy and in those who present with cryptococcal meningitis as the first manifestation of their HIV infection. He undoubtedly has a CD4 cell count of less than 200/L (perhaps even less than 50/L). Patients with AIDS tend to have a huge burden of cryptococcal organisms and a minimal host response, as shown by the cerebrospinal fluid findings in this patient. Increased intracranial pressure due to cryptococcal meningitis is found most often in patients with HIV infection but may also occur in patients with cryptococcal meningitis who do not have AIDS. Therefore, measurement of opening pressure at the time of lumbar puncture is especially important. The postulated mechanisms of increased intracranial pressure include cerebral edema secondary to the osmotic effect of the huge amount of polysaccharide capsular material as well as diminished absorption of cerebrospinal fluid because of plugging of the arachnoid villi with the same capsular material. Increased intracranial pressure is associated with a high mortality rate in AIDS patients who have cryptoccocal meningitis. The high mortality rate is directly related to the height of the pressure. Measures to decrease the pressure should be instituted immediately, preferably by daily lumbar punctures in order to drain enough cerebrospinal fluid to normalize the pressure. Some patients may require a ventriculoperitoneal shunt. However, this is not an emergent procedure and should not be done before antifungal agents are started. Although both should be given, neither antiretroviral agents nor antifungal agents will lower the intracranial pressure. The use of corticosteroids in this setting is controversial and may be associated with a poorer outcome. Infectious Disease Medicine:Question 6 The correct answer is C Educational Objectives Recognize the diagnosis of common variable hypogammaglobulinemia in a patient with multiple episodes of bacterial pneumonia. Critique The most likely diagnosis of those given is common variable hypogammaglobulinemia. The clinical picture is typical, and the serum protein electrophoresis results are suggestive, although not conclusive, for this disorder because of the low gamma globulin values. The remaining laboratory results are not consistent with any of the other diagnoses. The normal total hemolytic complement (CH50) value effectively rules out terminal complement component deficiencies, and, in any event, such deficiencies would not be expected to lead to bacterial pneumonia. Although bronchogenic carcinoma is a cause of postobstructive pneumonia, this diagnosis is not suggested by the radiographic findings and time course of this patients disorder. Multiple myeloma is usually, although not always, accompanied by a monoclonal spike in the gamma globulin region. Some patients with a predominance of light chain production may have a normal serum protein

electrophoresis. Since none of the tests performed to date definitively rules out any of the potential diagnoses, quantitative serum immunoglobulin determination should be done next. Infectious Disease Medicine:Question 7 The correct answer is D Educational Objectives Recognize the clinical manifestations of sepsis and differentiate this from systemic inflammatory response syndrome, severe sepsis, bacteremia, and septic shock. Critique Although this patient has documented bacteremia (defined as bacteria present in the blood and confirmed by culture), she also has evidence of a systemic response to infection (fever, tachycardia, tachypnea, and an elevated leukocyte count with immature band forms). She does not have organ dysfunction or perfusion abnormalities, which occur in patients with severe sepsis or septic shock. Therefore, the term that best defines her illness is sepsis. The American College of Chest Physicians/Society of Critical Care Medicine (ACCP/SCCM) Consensus Conference definitions of systemic inflammatory response syndrome, sepsis, severe sepsis, septic shock, and multiple organ dysfunction syndrome are as follows: Systemic inflammatory response syndrome (SIRS): The systemic inflammatory response to a wide variety of severe clinical insults, manifested by at least two of the following conditions: temperature greater than 38.0 C (100.4 F) or less than 36.0 C (96.8 F), heart rate greater than 90/mm, respiration rate greater than 20/mm or arterial blood Pco2 less than 32 mm Hg, leukocyte count greater than 12,000/L or less than 4000/L or with greater than 10% immature band forms. Sepsis: The systemic inflammatory response to a documented infection. In association with infection, manifestations of sepsis are the same as those described for SIRS. Severe sepsis: Sepsis associated with organ dysfunction, hypoperfusion, or hypotension. Septic shock: A subset of severe sepsis, defined as sepsis-induced hypotension despite adequate fluid resuscitation plus the presence of perfusion abnormalities. Patients receiving inotropic or vasopressor agents may no longer be hypotensive by the time they develop hypoperfusion abnormalities or organ dysfunction; however, they would still be considered to have septic shock. Multiple organ dysfunction syndrome (MODS): Presence of altered organ function in an acutely ill patient such that homeostasis cannot be maintained without intervention. Infectious Disease Medicine:Question 8 The correct answer is B Educational Objectives Understand how to diagnose and treat native valve endocarditis. Critique The patient meets both major Duke criteria for definite endocarditis (that is, a typical microorganism grown on two blood cultures and evidence of endocardial involvement met either by echocardiography because of the oscillating intracardiac mass or by physical examination because of the new aortic regurgitant murmur). His history also raises suspicion of endocarditis. Although not necessary to meet the diagnostic criteria in this case, the presence of the bicuspid aortic valve, fever, and conjunctival hemorrhage (but not the splinter hemorrhages) fulfills three of the minor Duke criteria for endocarditis. Endocarditis due to sensitive viridans streptococci on native valves can be treated for 4 weeks with penicillin or ceftriaxone or for 2 weeks when either agent is combined with synergistic low-dose gentamicin. In a patient with uncomplicated endocarditis, the addition of gentamicin decreases the total treatment course from 4 weeks to 2 weeks. In the absence of penicillin allergy or penicillin resistance, vancomycin is inappropriate.

Although adding synergistic doses of gentamicin is appropriate, using this combination regimen for 4 (or 6) weeks is not part of a standard treatment regimen. Similarly, ceftriaxone can be substituted for penicillin in a standard 4-week regimen but would not be used for 8 weeks for a patient with uncomplicated endocarditis. Infectious Disease Medicine:Question 9 The correct answer is C Educational Objectives Know the prophylactic management of persons in contact with a patient with possible smallpox. Critique The diagnosis of smallpox must be considered in this case. Vaccination with vaccinia can modify or prevent disease in contacts of this patient if given within several days of exposure to smallpox. One should not wait for confirmatory evidence of smallpox infection because confirmation may take too long to allow effective prevention. Vaccinia immune globulin has no effect on smallpox. Ribavirin also has no known effect on smallpox. Vaccinia immune globulin and vaccine should not be administered concurrently, as this may negate the protective effects of the vaccine. The health care workers should not be sent home at this time. Furlough is not recommended by the Centers for Disease Control and Prevention for health care workers who receive vaccinia vaccine. However, if they do not receive the vaccine, they should remain relatively isolated at home for the usual incubation period (7 to 17 days) in order not to expose others to the disease if they possibly become infected.

Infectious Disease Medicine:Question 10 The correct answer is C Educational Objectives Select the correct medication for chemoprophylaxis of malaria. Critique There are several effective forms of malaria chemoprophylaxis for the protection of travelers to areas of risk within sub-Saharan Africa. Currently available medications include mefloquine, doxycycline, and atovaquone/proguanil (Malarone). The efficacy of these agents is similar, and the choice is based primarily on a consideration of convenience, cost (atovaquone/proguanil is the most expensive), and likely adverse effects. Each chemoprophylactic regimen must begin prior to entry into the malaria-risk area and be continued for 1 week (atovaquone/proguanil) to 4 weeks (doxycycline and mefloquine) after leaving the area. Chloroquine resistance is widespread. As a result, chloroquine chemoprophylaxis is still effective in only a few areas of the world (for example, Central America, the Caribbean, North Africa, and parts of China). Chloroquine is not an appropriate prophylactic agent for the prevention of malaria in any area of sub-Saharan Africa. In some instances, quinine can be used for the treatment of malaria, but this drug is not currently recommended for chemoprophylaxis. Metronidazole and trimethoprim/sulfamethoxazole have no significant anti-Plasmodium activity and therefore play no role in the chemoprophylaxis of any form of malaria. Infectious Disease Medicine: Question 11 The correct answer is E Educational Objectives Understand the pathophysiology of IgA deficiency Critique

Although this patient has IgA deficiency, this disorder alone is not generally associated with multiple bacterial infections. The presence of low levels of one or more of the four lgG subsets is the most likely explanation for his increased bacterial infections, and serum lgG subset quantification should therefore be obtained. Measurement of total hemolytic complement (CH 50) is unlikely to be helpful, since the complement deficiencies associated with multiple pyogenic infections are quite rare (with the exception of neisserial infections). Lymphocyte disorders that cause abnormal lymphocyte subset quantification are also unlikely to be associated with enhanced susceptibility to bacterial infections. Repeat quantitative immunoglobulin determination is unnecessary because of the small likelihood of laboratory error, especially since the lgG and 1gM values are normal or high normal. Since IgA was not detectable, serum IgA subset quantification cannot be done. Also, even the documentation of low or absent values for only one of the two IgA subsets would not satisfactorily explain this patients increased number of infections. Infectious Disease Medicine:Question 12 The correct answer is C Educational Objectives Understand the importance of partner networks in the transmission of sexually transmitted diseases. Critique Although the prevalence of syphilis transmission is currently low in the United States, there are populations in whom the disease is still relatively common. In these groups involving certain sexual partner networks, the probability of acquiring syphilis from any given sexual encounter is many fold higher than in the general population. The sexual partner networks have been described in the context of Internet chat rooms, but they also apply to populations of people with multiple sexual partners within a limited demographic setting. HIV infection can be subtle, but a negative HIV test in the past year makes HIV-related syphilis extremely unlikely. It is, however, prudent to test all patients with newly diagnosed syphilis for HIV infection. Although occult chlamydial infections may occur in men or women, these infections do not seem to be related to the acquisition of syphilis. Gonorrhea is the most common setting for such concomitant chlamydial infections. When syphilis treatment is incomplete, the manifestations are of secondary or tertiary syphilis. To date, there are no penicillin-resistant strains of syphilis. Although testing of clinical specimens for resistance is not available, in vitro testing and clinical experience support this conclusion. Infectious Disease Medicine:Question 13 The correct answer is B Educational Objectives Recall the most appropriate therapy for early Lyme disease. Critique The patient has a classic history of early Lyme disease. She had a tick bite while in an endemic area, followed by a lesion consistent with erythema migrans. At this stage of the illness, the diagnosis is based on clinical guidelines alone, as 70% to 80% of patients with early disease will have a negative B. burgdorferi titer. Consequently, treatment should not await serologic confirmation. Amoxicillin is the most appropriate therapy for this patient. It is as effective as cefuroxime-axetil but is significantly less expensive. Doxycycline may be the treatment of choice in adults because it provides prophylaxis against other tick-borne infections (ehrlichiosis and Rocky Mountain spotted fever). However, the patient is allergic to this drug. Azithromycin is less effective than amoxicillin and should only be used in patients who are intolerant of amoxicillin, cefuroxime, and doxycycline.

Although it is possible that the patient is co-infected withEhrlichia or Babesia, it would be inappropriate to test for them at this time. In the absence of clinical symptoms, these titers are unlikely to be positive or, if positive, may indicate past infection. Infectious Disease Medicine:Question 14 The correct answer is A Educational Objectives Recall the management of an AIDS patient presenting with a solitary brain lesion. Critique The dilemma in managing this AIDS patient, who is at risk for all major AIDS-related complications, is that he has a single mass lesion of the brain, whereas at least two thirds of patients with toxoplasmic encephalitis have multiple ring-enhancing lesions. His presentation is therefore somewhat atypical. However, he does have other features that are more typical of toxoplasmosis: 1) the lesion arises from the basal ganglia, and 2) serologic studies indicate past infection consistent with reactivation of quiescent infection that occurs with AIDS-related immunodeficiency. The most appropriate management is to treat this patient presumptively for toxoplasmosis with pyrimethamine plus either sulfadiazine or clindamycin and to monitor his clinical and radiographic treatment response. Biopsy of the lesion is indicated only if he does not respond to initial treatment. Primary central nervous system lymphoma is a possible diagnosis for which lumbar puncture can be done to detect Epstein-Barr virus DNA. This test is reported to be both sensitive and specific for lymphoma in this setting. However, lumbar puncture is contraindicated in this patient because of the significant mass effect seen on the MRI scan. Cytomegalovirus encephalitis would not present as a mass lesion, and a bacterial brain abscess is no more common in AIDS patients than in other hosts. If his CD4 cell count were considerably higher than 200/L, a bacterial abscess would be more likely.

Infectious Disease Medicine:Question 15 The correct answer is A Educational Objectives Determine the role of various antibiotics in the treatment of resistant Acinetobacter infection. Critique Resistant gram-negative bacillary infections are common in very ill hospitalized patients. Many of these infections begin by colonization of patients in intensive care units. Each hospital tends to have certain organisms that predominate, although these change over time. Acinetobacter baumannii is seldom found in ambulatory patients but is one of the most commonly isolated gram-negative organisms in some hospitals. Differentiating between colonization and infection can sometimes be difficult, since infection is almost always preceded by a variable duration of colonization. However, not all patients who are colonized will become infected. Some Acinetobacter strains are broadly antimicrobial susceptible, but many are multidrug resistant and difficult to treat. Usually the potent carbapenems (for example, imipenem and meropenem) and some of the aminoglycosides (for example, amikacin and tobramycin) are most active in vitro. Some of the -lactamase inhibitors (especially sulbactam) also demonstrate activity. Cephalosporins such as cefepime and fluoroquinolones such as levofloxacin are usually ineffective. Although there is no substitute for knowing all in vitro data, early use of an active drug is strongly recommended when an infection is highly likely. The potent carbapenems are usually equally active against Acinetobacter, with the exception of ertapenem. Ertapenem is a once-a-day carbapenem with activity similar to other drugs in that class except that the spectrum does not include Pseudomonas

aeruginosa, Acinetobacter species, and some gram-positive bacteria. The usefulness of aminoglycosides alone or in combination has not been resolved for treating Acinetobacter pneumonia. Resistance to these drugs is variable, although resistance to amikacin is fairly low. Aminoglycosides may not be well suited for treating lung infections despite having good minimal inhibitory concentrations. A combination of drugs may be better for the most critically ill patients, but this issue is unresolved at present. Infectious Disease Medicine:Question 16 The correct answer is E Educational Objectives Understand how to suppress recurrent genital herpesvirus infections. Critique Frequent recurrences of genital herpes can be effectively prevented by chronic suppressive therapy with an oral drug active that is against herpesvirus. Acyclovir (400 mg twice daily), famciclovir (250 mg twice daily), or valacyclovir (500 or 1000 mg daily) can be used. Chronic suppression is usually initiated when a patient has at least six episodes yearly. Most patients will experience some benefit, and 40% to 72% will be free of recurrences at 1 year. Chronic use of these drugs also suppresses asymptomatic virus shedding, so that transmission to others may theoretically be reduced. However, patients on suppressive regimens should be told that they may still transmit herpes simplex virus to their partners. Occasional interruption of therapy is reasonable in order to judge the pattern of recurrences, although there is no evidence that suppressive therapy will decrease the frequency of episodes after the drug is stopped. There currently are no data to suggest (and no theoretical basis to believe) that any regimen other than daily therapy indefinitely with a drug active against herpes simplex virus can suppress genital herpes.

Infectious Disease Medicine: Question 17 The correct answer is C Educational Objectives Recognize the indications for meningococcal post-exposure prophylaxis. Critique Neisseria meningitidisinfection is common in communal living settings such as dormitories. The organism is transmitted via close contact with an infected patients respiratory secretions. Post-exposure prophylaxis is recommended for household contacts and for those health care workers and other hospital personnel who had close contact with the patients respiratory secretions. Only the intern who intubated the patient without personal protective equipment is at increased risk of contracting meningococcal disease. The nurse, the radiology technician, and the chaplain did not have close contact with this patients respiratory secretions and do not require chemoprophylaxis. Infectious Disease Medicine:Question 18 The correct answer is D Educational Objectives Know when to suspect an endemic mycosis in a patient who does not live in an endemic area. Critique Patients are increasingly presenting with endemic mycoses, such as coccidioidomycosis and histoplasmosis, outside of endemic areas. This increase is related to more frequent travel to areas that are endemic for these fungi and to increased vocational or leisure activities that allow exposure to the organism. Simply residing in or even passing through an endemic area can lead to development of coccidioidomycosis. In contrast,

histoplasmosis usually requires participating in an activity related to disruption of the soil or spelunking for aerosolization of the organism and development of subsequent infection. Older adults appear to be at greater risk for developing more serious infection with Coccidioides immitis and are the fastest growing group reported with symptomatic coccidioidomycosis. Recognizing the manifestations of endemic mycoses and taking a complete travel history are essential for diagnosis and appropriate treatment. Hantavirus pulmonary syndrome is an acute severe pneumonia that occurs mostly among young Native Americans and other residents who are from southwest states adjoining the Four Corners area (Arizona, Colorado, New Mexico, and Utah) and who have come in contact with mice that carry the virus. Nocardiosis occurs mostly in immunocompromised hosts, although it can occur in normal hosts. There is no specific link to the Southwest. Although atypical mycobacterial infections cause pneumonia, findings are usually similar to those of tuberculosis in that they include upper lobe cavitary infiltrates in persons with underlying pulmonary disease or diffuse disease in persons who are immunocompromised. Finally, pulmonary infiltrates are extremely uncommon in patients with ehrlichiosis. Infectious Disease Medicine:Question 19 The correct answer is D Educational Objectives Select the appropriate empiric antimicrobial regimen for a patient with pneumococcal meningitis. Critique This patient has meningitis caused byStreptococcus pneumoniae. The infection likely originated in her middle ear, and she was placed on an inappropriate antimicrobial agent (ciprofloxacin) that has no significant activity against pneumococci.S. pneumoniae meningitis can be diagnosed presumptively based on the positive cerebrospinal fluid Gram stain, and antimicrobial therapy should be targeted at the causative organism. Empiric antimicrobial therapy for pneumococcal meningitis is vancomycin plus a thirdgeneration cephalosporin (either cefotaxime or ceftriaxone), pending in vitro susceptibility testing. If the pneumococci are highly resistant to penicillin or the cephalosporins, administering either penicillin or ceftriaxone alone may not achieve adequate cerebrospinal fluid concentrations to kill these organisms. Although vancomycin has good in vitro activity against resistant pneumococci, it should not be used alone for treating pneumococcal meningitis because of its unreliable penetration into the cerebrospinal fluid. The addition of ampicillin to vancomycin does not provide increased activity against S. pneumoniae. Infectious Disease Medicine:Question 20 The correct answer is C Educational Objectives Be aware that reactivation tuberculosis occurs early in many patients. Critique This patient requires a sputum smear and culture for acid-fast bacilli. The development of reactivation tuberculosis often follows closely after tuberculin skin test conversion. At least 50% of the lifetime risk of tuberculosis occurs within 1 or 2 years of conversion. Chemoprophylaxis is therefore desirable for anyone with known conversion, since the risk of tuberculosis (and possible transmission of tuberculosis to others) exceeds the risk of the medication. This patient had pneumonia that was suggestive of reactivation tuberculosis because of the upper lobe location and the failure to respond to antibiotics. Although aspiration pneumonia could account for the findings in this setting, the upper lobe is an uncommon site, since aspiration pneumonia tends to occur in lung segments favored by the gravitational movement of oral or gastric flora. Although swallowing disorders do predispose to aspiration, this patient has no clinical manifestations of such disorders and does not require a swallowing function evaluation.

Changing antibiotics is sometimes helpful if a resistant organism is a concern. However, levofloxacin is usually preferred to azithromycin for treating resistantStreptococcus pneumoniae, which this patient seems unlikely to have because of the slow progression of her infection. Levofloxacin is not very effective for treating aspiration pneumonia because of its limited activity against oral anaerobes. Repeating a positive tuberculin skin test is not necessary unless this is being done for the evaluation of anergy. Infectious Disease Medicine: Question 21 The correct answer is E Educational Objectives Recall the appropriate management of a pregnant woman with HIV infection. Critique Antiretroviral therapy during pregnancy has been proved to be unequivocally effective in reducing vertical transmission of HIV infection. When such therapy is combined with the judicious use of cesarean section, the infection rate for newborns in the United States has dropped below 3%. This patient has satisfactory CD4 cell counts and viral load levels and, if not pregnant, would not require antiretroviral therapy. However, treatment is indicated to further reduce the already low risk of vertical transmission. Despite her low viral load, this patient should receive zidovudine monotherapy, beginning between weeks 14 and 34 of pregnancy. Antiretroviral therapy should be avoided during the first trimester unless there is a compelling reason to begin (or continue) such therapy. This asymptomatic woman with early-stage infection has no indications for first-trimester therapy. Moreover, efavirenz has been shown to be teratogenic in animals. Nevirapine alone at the time of delivery is effective in reducing the vertical transmission rate but is not as effective as standard treatment begun earlier in the pregnancy.

Infectious Disease Medicine:Question 22 The correct answer is D Educational Objectives Recall the clinical presentation of methicillin-resistant Staphylococcus aureus in the community. Critique The best explanation for this patients persistent infection is the presence of resistant bacteria, most likely community acquired methicillin-resistant Staphylococcus aureus (CAMRSA). These strains have been in circulation for a few years and differ from hospitalassociated strains in several ways. Most important clinically is that they are not as broadly antibiotic resistant as the nosocomial strains and can usually be treated with a number of available oral agents. However, infections caused by CAMRSA can be very serious, and deaths have occurred in children and healthy adults from progressive infectionafter treatment is initiated. Clindamycin, tetracyclines, or trimethoprim/sulfamethoxazole is usually effective. Adding rifampin may also be helpful. When treating patients with severe infection caused by more broadly resistant CAMRSA, linezolid would be an expensive but reasonable alternative. Persistent skin infections can be worrisome because they sometimes are an indication of more serious deep soft-tissue infections such as fasciitis or myositis. Conventional cellulitis is often a mild to moderately severe disease, depending on the underlying health of the patient. Although cellulitis can resolve without treatment, it usually improves faster with appropriate antibiotics. The rate of resolution is variable, and therapy is sometimes changed because of concerns regarding the efficacy of the initial treatment. In this healthy young man, the skin lesion was actually progressing at a time when he

was taking antibiotics. This might be explained by erythema migrans except for the time of year, the presence of pain, and the rapid progression. Lyme disease usually presents with a progressive local rash, but the time course is much slower, and lesions are typically painless. Abscesses can complicate skin infections and may not resolve without drainage (either spontaneous or surgical). However, this patient had no evidence of abscess either at the time of initial presentation or later. Fasciitis is a serious, deep softtissue infection that can progress rapidly. However, fasciitis most often occurs in persons who are otherwise ill or weak (for example, diabetics, geriatric patients, injection drug users). Infection caused by flesh-eating bacteria, such as Streptococcus pyogenes, can be severe and rapid, but should either have begun to respond to these antibiotics (all S. pyogenes strains are susceptible to penicillins and most cephalosporins) or have caused a much more toxic presentation by this time. Infectious Disease Medicine:Question 23 The correct answer is B Educational Objectives Recall how to diagnose and treat herpes simplex virus encephalitis. Critique This patient has encephalitis (fever, confusion, and cerebrospinal fluid showing lymphocytic pleocytosis, elevated protein, and normal glucose). Moreover, the history suggests involvement of the temporal lobe, which is the area most often affected in patients with herpes simplex virus encephalitis. Therapy should not be delayed in order to confirm the etiologic diagnosis because survival in these patients is directly related to the level of consciousness at the onset of treatment. Intravenous acyclovir significantly reduces mortality from herpes simplex virus encephalitis and should be started promptly when this diagnosis is a serious consideration. The dose of acyclovir for this indication is 10 to 15 mg/kg every 8 hours, which is higher than that used for other infections caused by herpes simplex virus. Intravenous acyclovir at 5 mg/kg every 8 hours is used for systemic herpes simplex virus infections, but this dose is considered inadequate for central nervous system infections. Although a positive polymerase chain reaction test of cerebrospinal fluid may be used to confirm the diagnosis and a negative test may be an indication for discontinuation of empiric therapy, treatment must begin before this result is available. Similarly, therapy should not be delayed to obtain another imaging study. Although brain biopsy had been the standard approach to diagnosis, this invasive test is now reserved for patients who are not responding to therapy or have other indications for neurosurgery, such as the need to evaluate a mass that could be an abscess or a tumor. Infectious Disease Medicine:Question 24 The correct answer is A Educational Objectives Recall the most appropriate management for a patient with a low-risk tick bite. Critique It is tempting to consider any tick bite as a cause for intervention, since development of various tick-borne illnesses is possible. However, three key factors should be considered before initiating prophylaxis or early treatment following a tick bite: species of the tick, geographic location of acquisition, and duration of attachment. In this case, there was no uncertainty about the species of tick, since an experienced observer determined that it was a deer tick 4xodes scapularis, formerly called Ixodes dammini). New Jersey is a state with a high incidence of Lyme disease and a low to medium incidence of babesiosis and ehrlichiosis. However, in both humans and animals, the minimal time of tick attachment for transmission of Borrelia burgdorferi infection is 24 hours (and most likely 48 hours or more). By this time, the tick is usually engorged and is much easier to see. Ticks on parts of the body that are difficult for the patient to see are more likely to go unnoticed than are ticks on the arm.

Although all the oral regimens listed are fairly safe, they offer little advantage over observation, since this patient has an infinitesimally small risk of developing Lyme disease. When treatment is indicated, amoxicillin is a suitable agent and is the drug of choice for treating erythema migrans in children. Azithromycin is a less suitable alternative and has a higher failure rate than amoxicillin, doxycycline, or a cephalosporin. Many oral and parenteral cephalosporins are effective for the treatment of Lyme disease. If the duration of tick attachment is significant and exposure occurred in a highly endemic area, evidence shows that a single dose of doxycycline is effective in preventing most cases of Lyme disease. However, the high dose needed (200 mg) may provoke nausea. Infectious Disease Medicine:Question 25 The correct answer is B Educational Objectives Understand the more uncommon complement deficiencies associated with neisserial disease and which laboratory studies are most appropriate for diagnosing these disorders. Critique Complement component deficiencies of the alternative pathway components properdin and factor D are associated with susceptibility to meningococcal disease but not with low levels of total hemolytic complement (OH 50). This is also true of C9 deficiency, and, if given as a choice, determination of the serum C9 level would have been an acceptable answer. Because the likelihood of a complement abnormality increases substantially after each neisserial infection, most experts would order additional diagnostic studies of alternative pathway complement components. Determination of individual C5, 6, 7, and 8 levels is not needed because if any of these complement components had been low or absent, the CH5O value would have been significantly lower than normal for the laboratory where the test was done. Serum lgG subset quantification and T-cell subset quantification are incorrect because defects in lgG or T-cell subsets are not known to lead to increased susceptibility to neisserial infections.

Infectious Disease Medicine:Question 26 The correct answer is D Educational Objectives Recognize the appropriate use of adjunctive dexamethasone in patients with bacterial meningitis. Critique The rationale for use of adjunctive dexamethasone in patients with bacterial meningitis is to attenuate the subarachnoid space inflammatory response that results from antimicrobial agent-induced lysis of meningitis pathogens. Previous studies, predominantly in infants and children, have demonstrated that adjunctive dexamethasone is beneficial for patients with meningitis caused byHaemophilus influenzae type b and, if started along with or prior to parenteral antimicrobial therapy, may be beneficial for patients with pneumococcal meningitis. In a recently published trial in adults with bacterial meningitis, adjunctive dexamethasone was beneficial in the subset of patients with pneumococcal meningitis and was associated with a reduction in the number of patients who had an unfavorable outcome or who died. If used, adjunctive dexamethasone must be administered concomitant with or just prior to the first dose of antimicrobial therapy. Although dexamethasone has been shown to decrease penetration of antimicrobial agents into cerebrospinal fluid, this has not been associated with a worse outcome. Infectious Disease Medicine:Question 27 The correct answer is B

Educational Objectives Recall the treatment of Enterobacter cloacae infection. Critique Emergence of a derepressed mutant of Enterobacter cloacae is the most likely cause of this patients second positive tracheal culture. In view of the absence of new or progressive infiltrates on chest radiographs, nosocomial pneumonia due to this derepressed mutant is unlikely. Without an infectious etiology, drug-induced fever on an allergic basis is the most likely cause of his persistent fever. Gram-negative bacillary pneumonia is usually treated for 2 to 3 weeks, so that changing to another antibiotic to which the organism is susceptible is required. Trimethoprim/sulfamethoxazole should therefore be started. Imipenem should not be given, since it is a -lactam and may cause a cross-reaction in patients who have an allergic reaction to -lactams. Cefepime is also a -lactam and is not active against derepressed mutants. Metronidazole is only active against obligate anaerobes, such as Clostridium difficile. In addition metronidazole is inactive against facultative anaerobes, such asE. cloacae. Infectious Disease Medicine:Question 28 The correct answer is D Educational Objectives Recognize the pathogens that cause pneumonia in AIDS patients. Critique This patient has Strongyloides stercoralishyperinfection. Strongyloidiasis is a helminthic infection that causes chronic intestinal infection in normal hosts. For patients with various immunodeficiency disorders such as AIDS, infection withS. stercoralis can cause severe disease. In such patients, eggs laid by adult worms hatch within the gut and release filariaform larvae that can autoinfect the host. This results in disseminated bacterial infections secondary to carriage of gut bacteria to sterile sites and/or massive invasion of various organs (usually the lungs) by the larvae themselves, resulting in severe respiratory compromise. Most HIV-infected patients withS. stercoralis hyperinfection have late-stage disease. A low circulating eosinophil count (eosinopenia) is thought to be a poor prognostic finding. In general, the prognosis in this setting is poor. Ascaris lumbricoides helminths are macroscopic and are not encountered in this setting. Pneumocystis carinii organisms certainly do not look like the parasites shown.Cryptosporidium parvum is a nonhelminthic parasite that causes profound diarrhea in AIDS patients.Taenia solium is the pork tapeworm whose major extraintestinal complication is focal brain lesions (cysticercosis). Infectious Disease Medicine:Question 29 The correct answer is C Educational Objectives Understand how to prevent influenza in a high-risk unvaccinated patient when an influenza outbreak is occurring in the community. Critique Vaccination is the primary way to prevent influenza. However, after influenza has been documented in a community, unvaccinated individuals at high risk for complications of influenza or those who are caring for high-risk persons can begin chemoprophylaxis. If concomitant vaccination is performed, chemoprophylaxis need only be continued for 2 weeks, by which time adults are presumed to have developed protective antibody levels. The U.S. Food and Drug Administration has approved amantadine and rimantadine for prophylaxis of influenza A, and oseltamivir (but not zanamivir at the time of this writing) for prophylaxis of influenza A and B. Administration of vaccine alone is not sufficient if influenza is already occurring in the community. This patient is at risk for acquiring influenza virus, developing influenza, and dyingall before immunity develops from the vaccine. There is no role for surveillance

influenza cultures in deciding how to manage such a patient. If vaccine is administered, chemoprophylaxis is given for 2 weeks rather than 6 weeks. However, if vaccination is not possible or if the vaccine is not expected to protect against the current virus strain, chemoprophylaxis can be used for a longer course (that is, for the duration of influenza activity in the community). Because influenza can persist in an area for several weeks, vaccination is useful even after influenza has been documented in a community. Infectious Disease Medicine:Question 30 The correct answer is B Educational Objectives Understand the reasons for the current practice of recommending antifungal treatment for a patient with candidemia. Critique Candida infections are the fourth most common cause of nosocomial bloodstream infections. The source is usually from the patients gastrointestinal tract or from a central venous catheter. Patients in surgical intensive care units are at great risk for candidemia because of surgical procedures involving the gut, the frequent need for parenteral nutrition supplied through a central venous catheter, the frequent occurrence of acute renal failure requiring hemodialysis, and the use of broad-spectrum antibiotics. This patient requires immediate treatment with fluconazole to prevent development of a focal infection. Candida should never be assumed to be a contaminant in a blood culture. Therefore, removing the catheter alone is not adequate therapy. Even transient candidemia can lead to endophthalmitis, vertebral osteomyelitis, or other serious focal Candida infections. Focal infections such as chorioretinitis may take weeks to develop, and treatment should not be withheld until such an infection is documented. The fact that only two of four culture bottles grew Candida reflects the obligate aerobic nature of the organism; the bottles incubated anaerobically will not support the growth of Candida. Fluconazole, 400 mg daily, is appropriate for treating candidemia in patients with normal renal function and should be adequate for treating most Candida bloodstream infections. If the organism is identified as Candida krusei or Candida glabrata, therapy should be changed to amphotericin B or caspofungin.

Infectious Disease Medicine:Question 31 The correct answer is A Educational Objectives Be aware of the role of alcohol and impulsivity in the transmission of sexually transmitted diseases. Critique Many strategies have been suggested to prevent sexually transmitted diseases. Aside from abstinence, none of them has been completely effective. There is information, however, that impulsive behavior can increase the risk of acquiring sexually transmitted diseases. The most common extrinsic reasons for impulsive behavior relate to alcohol and drug use. Reducing alcohol consumption in adolescents and young adults can lower the risk of sexually transmitted diseases. While it may be possible to ask potential sexual partners if they have any sexually transmitted diseases, it is unlikely that anyone will offer this information even on direct inquiry. And given the possibility of asymptomatic infection with Neisseria gonorrhoeae, even an honest answer may be inaccurate. It is reasonable to discuss risks for more serious sexually transmitted diseases such as HIV infection, although using barrier precautions is still prudent since some people are not aware of their HIV status. Hygiene measures such as showering are reasonable, but they have not been found to reduce the incidence of sexually transmitted diseases. Several decades ago, prophylactic antibiotics were shown to reduce the incidence of gonorrhea in military personnel such as sailors on

shore leave. There is currently no information about prophylaxis for gonorrhea. This is an important issue because drug resistance in strains of N. gonorrhoeae has increased markedly. Infectious Disease Medicine:Question 32 The correct answer is C Educational Objectives Recall management strategies for the prevention of recurrent urinary tract infections. Critique Antimicrobial agents can be useful in preventing recurrences of urinary tract infections. Many drugs have been tested, and the best outcomes are associated with trimethoprim/sulfamethoxazole, nitrofurantoin, and fluoroquinolones. A patient-initiated fluoroquinolone given for several days is effective in the early treatment of urinary tract infections. Most women are able to distinguish urinary tract infections from other illnesses with a high degree of accuracy. This is a cost-effective treatment that has the least effect on bacterial ecology. The use of -lactams is disappointing for prophylaxis because gastrointestinal flora can be quickly altered so that -lactam-resistant organisms proliferate. This, in turn, can cause recurrent urinary tract infections even during periods of treatment or prophylaxis. Use of trimethoprim/sulfamethoxazole for prophylaxis may be of concern because the number of strains of uropathogens resistant to this agent has been gradually increasing. Furthermore, the dose for prophylaxis should be lower than the full therapeutic dose in order to minimize the risk of side effects that are somewhat dose- and durationdependent. Spermicides increase the risk of urinary tract infections. Therefore, patients with recurrent infections should be advised to use some other form of contraception. Factors such as sexual positioning, post-coital voiding, and post-toileting habits have not been shown to correlate with the risk of developing acute or recurrent urinary tract infections.

Infectious Disease Medicine:Question 33 The correct answer is C Educational Objectives Recognize the indications for empiric contact isolation for a patient with probableClostridium difficile infection. Critique This patient is at high risk forCiostridium difficile infection on the basis of her hospitalization and treatment with antibiotics. In high-risk patients with suspectedC. difficile infection, empiric contact isolation should be instituted until the pathogen can be ruled out, asC. difficile is easily spread by contact and may cause outbreaks of nosocomial diarrhea. Routine bacterial cultures are rarely helpful, since they primarily determine foodborne pathogens such as Salmonella, Shigellaand Campylobacter, which are all uncommon in hospitalized patients. Antimotility agents are contraindicated in patients withC. difficile diarrhea. Because the patient has a soft abdomen and normal bowel sounds, radiologic imaging is not indicated. Treatment of C. difficile diarrhea requires discontinuing the causative antibiotic and starting metronidazole or vancomycin. Adding an aminoglycoside without stopping the cephalosporin provides no benefit. Infectious Disease Medicine:Question 34 The correct answer is D

Educational Objectives Understand how to manage Staphylococcus aureus bacteremia. Critique The patient has an intravascular catheter-related bloodstream infection. Although the focus of infection was removed, the optimal duration of treatment in this setting is uncertain. Perhaps 25% of such patients have endocarditis, which can be detected most reliably by transesophageal echocardiography. Two studies suggest that the most costeffective approach for patients with native valves is to obtain a transesophageal echocardiogram and then treat for 4 weeks if endocarditis is diagnosed but for only 2 weeks if endocarditis is not diagnosed. However, some clinicians may want to treat conservatively for 4 (or even 6) weeks without performing an echocardiogram. Others may choose to be guided by their clinical assessment (with or without a less invasive, but less sensitive, transthoracic echocardiogram) and consider stopping the intravenous antibiotics after 2 weeks, although published reports suggest that clinical judgment may not be reliable in this setting. Of the choices given, the only option that falls within these parameters is obtaining a transesophageal echocardiogram and treating with intravenous nafcillin for 14 days if this study does not show endocarditis. Seven days of intravenous antibiotics would generally not be considered sufficient forS. aureus bacteremia, although some investigators have used such a short course in selected patients who have a normal transesophageal echocardiogram. However, 8 weeks would be an unnecessarily long antibiotic course, given the current information. Changing to an oral antibiotic is not considered a reliable option for treating S. aureus bacteremia, although some investigators have used a combination of oral rifampin and a fluoroquinolone. Finally, if a transesophageal echocardiogram is done and does not show endocarditis, there is no reason to provide treatment for 6 weeks. Infectious Disease Medicine:Question 35 The correct answer is C Educational Objectives Recall the treatment of extended-spectrum -lactamase Escherichia coli infection. Critique Klebsiella pneumoniae and Escherichia coli strains that are resistant to ceftazidime should be suspected of producing an extended-spectrum -lactamase (ESBL) that is capable of inactivating all extended-spectrum cephalosporins, penicillins, and aztreonam despite in vitro assay results to the contrary. Infections caused by these organisms may be unresponsive to the antibiotics listed. Hyperproduction of ESBLs may overcome inhibition by -lactamase inhibitors, such as tazobactam, and may inactivate cefepime, which is usually more resistant to -lactamases than are other extended-spectrum cephalosporins. The carbapenems are usually active against these organisms. Therefore, the most appropriate therapy for this patient is treatment with a carbapenem such as imipenem. The other options are inappropriate because many of these strains are resistant to multiple chemically unrelated classes of antimicrobial agents, such as aminoglycosides, tetracyclines, fluoroquinolones, and trimethoprim/sulfamethoxazole. Infectious Disease Medicine:Question 36 The correct answer is E Educational Objectives Recognize immune reconstitution syndrome in a patient with late-stage HIV infection who recently began taking antiretroviral therapy. Critique This patient presents with paradoxical worsening of his tuberculosis following the initiation of antiretroviral and antituberculous therapy. Such reactions are thought to be due to partial immune reconstitution, which occurs following effective control of HIV by antiretroviral therapy. This type of immune reconstitution syndrome reaction has also

been described in patients with other AIDS-related opportunistic infections, including disseminated Mycobacterium avium complex infection and cytomegalovirus retinitis. This patients findings are unlikely to be due to failure of either treatment regimen. His current treatment should therefore be continued along with the addition of a nonsteroidal anti-inflammatory drug for control of pain and fever. Lymph node biopsy and diagnostic thoracentesis are unlikely to be harmful, but are probably unnecessary at this time. Although not listed as an option, some clinicians advocate administering short, tapering courses of corticosteroids is advocated by some clinicians. To date, however, no large published series have addressed the safety or efficacy of this approach. Infectious Disease Medicine:Question 37 The correct answer is E Educational Objectives Understand the management of herpes labialis in an otherwise healthy individual. Critique Recurrent herpes labialis is a self-limited illness for which interventions are probably of marginal value. Use of topical penciclovir was found to have small but statistically significant benefits in the management of herpes labialis (time to healing and loss of pain were reduced by less than 1 day when patients began application within 1 hour of the first sign or symptom and applied the drug every 2 hours while awake for 4 days). An over-the-counter cream containing a long-chain alcohol, docosanol (Abreva ), is also FDA-approved for herpes labialis and has benefits similar to those of penciclovir when started within hours of the onset of symptoms. Recently, valacyclovir was approved for 1-day treatment of orolabial herpes, using two large oral doses 12 hours apart. Another possibility is to prescribe oral acyclovir in an attempt to suppress frequent recurrences or prevent sun-induced episodes of herpes labialis. Although there is support in the literature for this approach, acyclovir is not FDA-approved for this indication. A culture to confirm the diagnosis of uncomplicated herpes labialis is unnecessary. Antibacterial agents are not indicated for treatment of viral infections and should not be used to prevent bacterial superinfections. No treatment would be expected to help if started the day after the lesion was noted.

Infectious Disease Medicine:Question 38 The correct answer is B Educational Objectives Recognize the clinical presentation of hemorrhagic viral illness and some of the other illnesses to be included in the differential diagnosis. Critique Neisseria meningitidi Ebola virus, and Marburg virus are all capable of causing this patients clinical picture. Therefore, he should be treated with ceftriaxone or cefotaxime to cover the possibility of meningococcal meningitis. Specific treatment is not available for the two possible viral infections. Infection control measures should include contact and droplet isolation procedures, at least until the diagnosis is known. The clinical picture is not typical of plague ersinia pestis) in either of its major forms, bubonic or pneumonic. Infectious Disease Medicine:Question 39 The correct answer is A Educational Objectives Recall the complement component deficiencies that may be associated with increased pyogenic infections. Critique C3 deficiency is one of the most severe complement component deficiencies because it is

associated with large numbers of serious pyogenic infections. A total hemolytic complement (CH50) level would be expected to be low in the presence of a low to absent C3 value. Properdin and C5 deficiencies are associated only with increased numbers of neisserial infections. The patients IgA value was previously normal, so there is no need to repeat this study. Cl inhibitor (Cl INH) deficiency is the cause of hereditary angioedema and is not known to be associated with increased numbers of infections. Infectious Disease Medicine:Question 40 The correct answer is A Educational Objectives Recall the treatment of levofloxacin-resistant Streptococcus pneumoniae infection. Critique Streptococcus pneumoniae that is resistant to penicillin is usually resistant to multiple other chemically dissimilar antibiotics, such as the macrolides and trimethoprim/sulfamethoxazole, and may also be resistant to clindamycin, streptogramin b (a component of quinupristin/dalfopristin), and doxycycline. These strains exhibit no predictable cross-resistance to vancomycin, fluoroquinolones, or linezolid. However, this patient has likely developed levofloxacin resistance because of inadequate serum concentrations of levofloxacin relative to the minimal inhibitory concentration of the pathogen. Such strains may exhibit cross-resistance to all the fluoroquinolones. The most appropriate antibiotic for this critically ill patient is intravenous vancomycin, asS. pneumoniae resistance to this agent has not been reported to date. Resistance has been reported to the other antibiotics.

Infectious Disease Medicine:Question 41 The correct answer is C Educational Objectives Recognize long-term complications of HIV antiretroviral therapy. Critique With the advent of highly active antiretroviral therapy came the recognition of a fat redistribution syndrome termed HIV lipodystrophy. The mechanism for this disorder is not well understood but may involve a complex interplay between viral infection and antiretroviral therapy. Protease inhibitors are most closely associated with this syndrome, but nucleoside analogues, most notably stavudine, have also been implicated. The syndrome has also developed in patients who have never received antiretroviral therapy. HIV lipodystrophy may be associated with hyperlipidemia and glucose intolerance. Findings of lipodystrophy consist of some combination of fat wasting, primarily involving the face and extremities (lipoatrophy), with fat accumulation in the abdomen (visceral adiposity), breasts (in women), and sometimes in the face and dorsum of the neck (fat accumulation in the face and neck may mimic features of Cushings syndrome). No effective treatment has been identified to date. There are no findings at this time to support a diagnosis of lymphoma in this patient. HIV wasting is a late-stage complication that is associated with profound and progressive weight loss and cachexia. Although chronic hepatitis C is a possibility, hepatitis would not explain all of this patients physical findings. Infectious Disease Medicine:Question 42 The correct answer is B Educational Objectives Understand the emerging epidemiology of aspergillosis in a hematopoietic stem cell transplant patient and the usefulness of high-resolution CT scanning in helping to define the probability of invasive aspergil losis. Critique

This woman has symptoms strongly suggesting invasive pulmonary aspergillosis for which amphotericin B is indicated. Her risk factors include leukemia, hematopoietic stem cell transplantation, prior neutropenia, prior use of broad-spectrum antibiotics for bacterial infections, and, most importantly, chronic graft-versus-host disease treated with high-dose corticosteroids. Recent data suggest that most transplant recipients with invasive aspergillosis do not develop infection in the pre-engraftment phase; instead, infection occurs when they develop graft-versus-host disease that is treated with highdose corticosteroid therapy. High-resolution CT scanning frequently allows visualization of multiple pulmonary nodules that are not seen on chest radiographs. Scans also frequently show a pulmonary infiltrate with a ground-glass appearance around the nodule, termed a halo sign. The halo sign indicates bleeding and is characteristic of changes induced by angioinvasive fungi, such asAspergillus. High-resolution CT scanning of the chest should be followed by bronchoscopy to identify the exact etiologic agent, since several angioinvasive fungi have the same clinical and radiologic features. The patients presentation is not typical of a catheter-related infection; therefore, the central venous catheter should not be removed. Fluconazole is not effective for aspergillosis, and there is a high likelihood that the patient was, and may still be, on fluconazole prophylaxis. Atypical pneumonia such as Legionnaires disease is not usually associated with hemoptysis. Therefore, azithromycin is not indicated. Infectious Disease Medicine:Question 43 The correct answer is C Educational Objectives Understand key aspects of antibiotic management when treating gram-positive prosthetic valve endocarditis. Critique Many laboratories test enterococci for high-level resistance against gentamicin and streptomycin. Lack of high-level resistance correlates with synergistic killing when the aminoglycoside is added to penicillin, ampicillin, or vancomycin (assuming that the bacteria are also sensitive to these agents). When gentamicin is used as synergistic therapy for gram-positive infections, the starting dose is only 1 mg/kg of body weight every 8 hours (with adjustments for obesity and renal function), and the goal is to obtain peak levels just over 3 ig/mL. This dose maximizes the clinical benefit while minimizing nephrotoxic side effects. Higher peak levels are considered therapeutic only when aminoglycosides are used to treat gram-negative rod infections. As with all aminoglycoside regimens, trough levels should be low to reduce the risk of toxicity. For this patient, no adjustment in either medication is necessary. For the reasons discussed above, there is no reason to increase the gentamicin dose, despite being informed that the level is subtherapeutic according to the laboratorys guidelines. Similarly, there is no reason to decrease the interval between gentamicin doses, which is another way to increase the total daily dose. The vancomycin levels are appropriate, and there is no reason to change that dose. The standard duration of treatment for prosthetic valve enterococcal endocarditis is 6 weeks of combination therapy, although a recent report questioned the need to continue the aminoglycoside beyond 2 weeks. Infectious Disease Medicine:Question 44 The correct answer is C Educational Objectives Select the appropriate antimicrobial agent for a patient with Listeria meningitis who has a severe allergy to penicillin. Critique This patient, who underwent renal transplantation and is on immunosuppressive therapy,

has meningitis caused byListeria monocytogenes She likely acquired the infection through the gastrointestinal tract, following consumption of cole slaw and/or processed meats, both of which are vehicles of transmission in outbreaks of Listeria infection. The grampositive bacilli on Gram stain of cerebrospinal fluid indicate that Listeria should be highly suspected as the causative organism. The therapy of choice for Listeria meningitis is ampicillin or penicillin G combined with an aminoglycoside. Given the patients severe allergy to penicillin, trimethoprim/sulfamethoxazole is the alternative antimicrobial agent of choice. Erythromycin, chloramphenicol, and vancomycin have in vitro activity againstL. monocytogenes but are associated with unacceptably high failure rates in patients withListeria meningitis. Levofloxacin has not been studied for treatment of meningitis. Infectious Disease Medicine:Question 45 The correct answer is B Educational Objectives Recognize the indications for yellow fever vaccination in a traveler. Critique Yellow fever is caused by a flavivirus transmitted by mosquitoes. The disease occurs only in tropical areas of Africa and South America. Yellow fever is a potentially life-threatening viral hemorrhagic fever with a case fatality rate of 25% to 50% and, as such, vaccination should be considered for all travelers to areas of risk. About 90% of individuals receiving the vaccine produce neutralizing antibodies within 10 days of immunization, and about 99% do so within 30 days. Serious adverse reactions to yellow fever vaccine are quite rare and include hypersensitivity reactions, encephalitis, and a multiorgan failure syndrome resembling naturally acquired yellow fever infection. Some surveillance data suggest that elderly persons may be more prone to severe reactions, but the evidence is not conclusive. With the exception of a minority of persons who have potential contraindications, yellow fever vaccination is recommended for all travelers to endemic areas. Potential contraindications include age less than 9 months, history of hypersensitivity to a component of the vaccine, immunosuppression, and pregnancy. For any person with a potential contraindication, the decision to vaccinate must be individualized and is based on the estimated risk of infection balanced by the potential severity of an adverse reaction. In such cases, consultation with an expert in the area of travelers health is usually recommended.

Infectious Disease Medicine:Question 46 The correct answer is D Educational Objectives Recall the most appropriate antibiotic therapy for osteomyelitis of the metatarsal head. Critique Linezolid plus imipenem is the most appropriate choice to provide coverage for all pathogens. Linezolid is active againstEnterococcus faecium and methicillin-resistant Staphylococcus aureus, and imipenem is active against gram-negative bacilli. Imipenem susceptibility testing does not predict ertapenem susceptibility forPseudomonas aeruginosa, Acinetobacter species, and enterococci, against which ertapenem is inactive. Quinupristin/dalfopristin is usually inactive against gram-negative bacilli. Imipenem is not active againstE. faecium, and all -lactams, including the carbepenems (imipenem, meropenem, and ertapenem), are inactive against methicillinresistant S. aureus Infectious Disease Medicine:Question 47 The correct answer is D Educational Objectives

Recognize the unique pharmacologic issues in the treatment of tuberculosis in an HIVinfected patient. Critique This patients presentation is most consistent with tuberculosis, given the radiographic features and the positive smear for acid-fast bacilli. Although Mycobacterium avium complex (MAC) infection is a well-known complication of AIDS, isolated MAC pneumonia is much less common than disseminated MAC infection. Furthermore, disseminated infection usually develops in patients with profoundly low CD4 cell counts (< 50/L). The risk of disseminated MAC infection is drastically reduced by immune reconstitution following highly active antiretroviral therapy (HAART). This patients HIV infection is under good (but not perfect) control after what appears to be several failed attempts with regimens that included nucleoside analogues and nonnucleoside reverse transcriptase inhibitors. There is no easy way to avoid continuing this patients protease inhibitor-based regimen. An antiretroviral genotype or phenotype determination is unlikely to be helpful, since his viral load is too low for such a measurement to be obtained. Rifampin is known to interfere with many protease inhibitors and should be avoided. Substituting rifabutin for rifampin is appropriate, pending identification and sensitivity testing of this patients sputum isolate. A streptomycin-containing regimen is a possibility but is likely to be more toxic. Waiting for culture results before starting therapy is inadvisable, since results can take many weeks and the patient may decline significantly or die in the interim. Infectious Disease Medicine:Question 48 The correct answer is D Educational Objectives Recall when and how to treat uncomplicated chickenpox (primary varicella). Critique This patient has the classic presentation of chickenpox (note that lesions at multiple stages on the same part of the body are not consistent with smallpox). Because lesions have been present for 4 days, the most appropriate management is to provide anticipatory guidance about complications of varicella and offer symptomatic treatment. When started within 24 hours of onset of rash, high-dose oral acyclovir can shorten the duration of fever by about 1 day and decrease the number of new chickenpox lesions in children, adolescents, and adults. However, no benefit was noted when adults started treatment 25 to 72 hours after the rash developed. There are also no data to support the use of topical or intravenous acyclovir in this setting. Clouding of the liquid within vesicles is expected as the lesions evolve, and antibacterial agents are not needed for these small pustules. In addition, antibiotics should not be given to prevent bacterial superinfections but instead should be reserved for established bacterial infections. Infectious Disease Medicine:Question 49 The correct answer is A Educational Objectives Recall the management of health care workers exposed to Mycobacterium tuberculosis. Critique This patient most likely has severe tuberculosis because of his history and the positive sputum smear. Although many health care workers do not develop tuberculosis even after close and long-term exposure, some exposed workers do contract the disease. Having a protocol for evaluating this possibility is therefore important. Most institutions have policies that require regular tuberculin skin testing. All employees who are exposed to patients with active tuberculosis should have repeat skin testing at the time of exposure and again in about 12 weeks. A persistently negative test result requires no further intervention. A positive test result

suggests conversion and should be followed by appropriate treatment or prophylaxis. In most cases, isoniazid alone for 9 months is adequate, but more than one drug may be necessary after exposure to a source patient with resistantMycobacterium tuberculosis. Therapy can be modified based on the results of cultures obtained from the source patient. Chest radiographs are useful in managing patients with clinical tuberculosis or with positive tuberculin skin tests. They have no role in screening if tuberculin skin testing has not been done. Infectious Disease Medicine:Question 50 The correct answer is E Educational Objectives Understand the manifestations of X-linked lymphoproliferative disease. Critique After infection with Epstein-Barr virus, persons with X-linked lymphoproliferative disease can have any of the findings noted in this patient, his older brother, and his maternal uncle.The febrile illness that this patient had several months ago was presumably infectious mononucleosis. Although finding positive Epstein-Barr virus antibody titers would not prove that he had infectious mononucleosis, the presence of these antibodies would provide support for this diagnosis. Definitive proof would require molecular techniques to demonstrate mutation of the SH2D1A gene that is responsible for the disorder. HIV can be associated with lymphoma and pancytopenias but is not known to cause profound hypogammaglobulinemia. Cytomegalovirus, human T-call lymphotropic virus-i (HTLV-i) and HTLV-2, and human herpesvirus 6 are not known to cause any of the findings identified in the patient, his brother, or his uncle. Infectious Disease Medicine:Question 51 The correct answer is D Educational Objectives Diagnose babesiosis in a patient co-infected with Lyme disease. Critique This patients current presentation is compatible with babesiosis. He also resides in an endemic area, had contact with an appropriate tick (as attested to by his prior bout of Lyme disease), and is asplenic. The diagnostic test for babesiosis is visualization of the causative organism on a peripheral blood smear. Babesiosis is often compared with malaria, since the single-cell organism for each infection is a parasite of the erythrocyte and both illnesses cause fever and hemolysis. Doing serologic studies for Lyme disease is not indicated, as results would be positive because of this patients prior exposure toBorrelia burgdorferi or because of his recent (or even a remote) episode of clinical Lyme disease. Alternatively, he may not make any antibodies toB. burgdorferi since treatment may have aborted the immunogenicity. Although the possibility of ehrlichiosis does exist in New England, there are several problems with this diagnosis. The time from exposure to disease development is much too long, the prior treatment with doxycycline should have eradicated all Ehrlichia organisms, and there should be no evidence of significant hemolysis. Finally, the tick that transmits Rocky Mountain spotted fever is different from the tick that transmitsB. burgdorferi. Infectious Disease Medicine:Question 52 The correct answer is C Educational Objectives Recall the management of a patient with latent tuberculosis infection. Critique Therapy for latent tuberculosis is indicated for this patient. She is from a developing country where tuberculosis is widespread and where she received bacille Calmette-Gurin

(BCG) vaccination. Any patient for whom tuberculin skin testing is indicated and who has a positive test result should be considered for chemoprophylaxis unless underlying medical conditions such as chronic liver disease make such treatment hazardous. The first-line regimen for chemoprophylaxis in all patients is isoniazid, 300 mg daily for 9 months. The history of BCG vaccination should be ignored when considering whether to recommend chemoprophylaxis because of the high prevalence of tuberculosis in this patients country of origin and the marginal effectiveness of the BCG vaccine. Although older guidelines suggested not using chemoprophylaxis for tuberculin skin test reactors older than 35 years of age because of the risk of hepatotoxicity, age is no longer a factor when deciding about therapy. Alternative regimens such as rifam pin and pyrazinamide for 2 months or rifam pin for 4 months should be considered only for patients who cannot tolerate isoniazid or who are likely to be infected with an isoniazid-resistant strain. Infectious Disease Medicine:Question 53 The correct answer is B Educational Objectives Recall the currently accepted management of a patient with septic shock. Critique This patient had septic shock from pneumococcal pneumonia with bacteremia. In addition to prompt use of antibiotic agents, treatment of septic shock continues to focus on eradicating infection and supporting failing organs. The decreased mortality rates in some subgroups of patients with severe sepsis suggest that improved basic supportive measures are beneficial. These include nutritional support, judicious use of sedation, and prevention of stress ulcers, skin breakdown, and deep venous thrombosis. Although nutritional support appears to be beneficial, the optimal level, route of delivery, timing, nutritional formula, and monitoring method are debated. There is no evidence that parenteral nutrition is superior to enteral feeding. Despite the encouraging results of some pilot studies, use of corticosteroids, nitric oxide, or N-acetylcysteine is not accepted therapy and is not indicated currently. Some experts administer colloid, but this has not been documented to be more effective than crystalloid fluid administration.

Infectious Disease Medicine:Question 54 The correct answer is E Educational Objectives Recall the treatment of enterococcal bacteremia secondary to a central intravenous catheter. Critique The patient most likely has enterococcal bacteremia secondary to a central intravenous catheter infection. Persistent bacteremia may be a consequence of either the intravenous catheter remaining in place, the enterococcus being vancomycin-resistant, or both. Linezolid is active against both Enterococcus faecium and Enterococcus faecalis and exhibits no cross-resistance with vancomycin. Therefore, linezolid is the most appropriate antibiotic in this situation. Enterococci are inherently resistant to clindamycin and are unpredictably susceptible to fluoroquinolones. Although most vancomycin-resistant enterococci are E. faecium, some are E. faecalis and quinupristin/dalfopristin is not active againstE. faecalis.

Infectious Disease Medicine:Question 55 The correct answer is A Educational Objectives Recognize the adverse effects of antiretroviral agents. Critique This patient presents with advanced HIV infection. Because of her low CD4 cell count and some symptoms, it is reasonable to begin antiretroviral therapy with a regimen such as the one used. However, the new symptoms that develop 1 week after starting treatment are consistent with abacavir hypersensitivity reaction. Such reactions have been fatal. Most of the fatalities have occurred in patients who restarted abacavir after temporarily stopping this agent until symptoms subsided. Therefore, abacavir should never be reinstituted, and substituting another drug for this agent is required. Zidovudine causes nausea and fatigue but has not been known to cause this patients other symptoms. Pneumocystis carinii pneumonia can cause a dry cough and fever but would not be expected to induce a rash. The patients CD4 cell count may be somewhat high for development of P. carinii pneumonia, although this can occur in patients with CD4 cell counts as high as 250/L. Infectious Disease Medicine:Question 56 The correct answer is B Educational Objectives Know the treatment of herpes labialis in an immunocompromised patient. Critique The patient has herpetic stomatitis due to herpes simplex virus, which appears to be mild at this point. Although only the topical and intravenous forms of acyclovir are approved by the U.S. Food and Drug Administration for treatment of mucocutaneous herpes simplex virus in immunocompromised patients, the oral form is a reasonable option for this patient. One study reported that oral acyclovir reduced the duration of various clinical end points by 5 to 13 days in bone marrow transplant recipients with this viral infection. The topical form may be used for limited skin disease, and the intravenous form must be used if the patient cannot take oral medications or if there is evidence of visceral involvement. Mupirocin is used for staphylococcal and streptococcal skin infections; there is no role for antibacterial agents in the treatment of viral infections. Ganciclovir is active against any virus for which acyclovir is used (and is also effective for cytomegalovirus), but ganciclovir is more toxic and more expensive than acyclovir. Although pain medication may be necessary as adjunctive therapy for patients with herpetic stomatitis, nothing in the clinical vignette suggests that pain relief is necessary for this patient. Corticosteroids are not indicated for treatment of herpes simplex virus infections and may exacerbate the disorder. Infectious Disease Medicine:Question 57 The correct answer is A Educational Objectives Be aware of changing patterns of resistance of uropathogens in the community. Critique In the United States, uropathogens have become increasingly resistant to trimethoprim/sulfamethoxazole over the past several years. This is variable by region but has been noted in persons utilizing student health services and is apparently caused in part by a nationally disseminated epidemic strain of resistant uropathogenicEscherichia coli. Limited improvement and early clinical failure are common markers of this problem. Nevertheless, some bacterial strains with in vitro resistance will respond to therapy, but the response is unpredictable and probably occurs in less than 50% of patients with these strains. Failure to complete therapy is associated with relapse and treatment failure. However, most women with susceptible organisms respond to very small doses of antibiotics. A

single-dose treatment regimen is often successful, although this approach is currently seldom used. Obtaining a urinalysis (or at the minimum a dipstick examination of the urine) is customary when a woman has symptoms suggesting a urinary tract infection. However, there is good evidence that women are usually able to identify such infections themselves - especially if they have had similar problems before. This allows the use of telephone-based prescribing as well as patient-initiated treatment of recurrent infections. Reinfection may possibly explain this patients recurrent symptoms, but reinfection rarely occurs when trimethoprim/sulfamethoxazole is used. Differentiating reinfection and new infection requires a urine culture at the start of treatment and at the time of apparent reinfection. Infectious Disease Medicine:Question 58 The correct answer is C Educational Objectives Recognize the indications for post-exposure prophylaxis for HIV. Critique After a high-risk exposure to blood (for example, a bloody, hollow needle) from a highrisk patient (for example, an injection drug user with multiple sexual partners), prevention of HIV transmission is paramount. Antiretroviral therapy should be initiated as quickly as possible, preferably within 20 minutes of the exposure. However, because all antiretroviral agents have potential side effects, therapy should be continued only if HIV is confirmed in the source patient. If the source patient is HIV negative, therapy should be discontinued. Because antiretroviral agents should be started as quickly as possible, waiting for the test results is inappropriate. HIV testing by polymerase chain reaction is not indicated for diagnosing HIV and has been associated with false-positive results. All persons with high-risk exposures should be treated with post-exposure prophylaxis rather than simply being followed by serologic testing. Infectious Disease Medicine:Question 59 The correct answer is B Educational Objectives Recall the initial therapy for a patient with a presumptive diagnosis of anthrax pneumonia with central nervous system involvement. Critique Because of the clinical presentation and cerebrospinal fluid findings, inhalational anthrax with central nervous system (CNS) involvement is the major diagnostic possibility. A combined regimen of ciprofloxacin or doxycycline plus clindamycin and vancomycin is most appropriate for treating the majority of known strains of anthrax with CNS involvement. Administration of ceftriaxone, ampicillin, and vancomycin is incorrect because cephalosporins are not active against anthrax. Although this would be a reasonable regimen for a patient with community-acquired meningitis or community-acquired pneumonia, this patient had gram-positive rods on a stained cerebrospinal fluid specimen and a clinical picture most consistent with anthrax pneumonia with CNS involvement. Administration of ceftriaxone, penicillin G, azithromycin, and rifampin is incorrect for the same reasons. Finally, giving just two agents, ciprofloxacin and doxycycline, would be inadequate, especially in a patient with CNS involvement. Infectious Disease Medicine:Question 60 The correct answer is C Educational Objectives Select the best imaging modality for diagnosing necrotizing fasciitis. Critique

Necrotizing fasciitis is a life-threatening infection that requires immediate surgery. The patient frequently presents with severe sepsis and pain that is disproportionate to the physical findings. Tense edema and bullous skin lesions are also frequent findings. MRI is the most sensitive imaging study for diagnosing necrotizing fasciitis, and the finding of high intensity of the fascia in T2-weighted images is highly suggestive of fasciitis. A plain radiograph is suitable for detecting gas in the tissues and is more sensitive than palpation. However, gas is not always present in necrotizing fasciitis. CT is sensitive for diagnosing an abscess or a fluid collection but is less sensitive than MRI for diagnosing fasciitis. Ultrasound examination is an insensitive study for detecting this disorder. Infectious Disease Medicine:Question 61 The correct answer is A Educational Objectives Recall the treatment of a patient with vancomycin-resistant and methicillin-resistant Staphylococcus aureus infection. Critique The plasmid that encodes for vancomycin resistance is usually found only in vancomycinresistant enterococci. However, acquisition of this plasmid by methicillinresistantStaphylococcus aureus (MRSA) has recently been reported in two patients in the United States. Prolonged and perhaps inadequate vancomycin treatment of these patients possibly allowed emergence of vancomycin-resistant MRSA. Administration of linezolid is the most effective treatment for this patient. Trimethoprim/sulfamethoxazole and clindamycin are not active against enterococci. Imipenem is a -lactam and is therefore not active against MRSA. Finally, quinupristin/dalfopristin is not active against Enterococcus faecalis (despite usually being active againstEnterococcus faecium). Infectious Disease Medicine:Question 62 The correct answer is A Educational Objectives Understand the treatment of primary genital herpesvirus infection. Critique The patient appears to have primary genital herpesvirus infection, which often is severe and may be associated with systemic symptoms. Treatment with acyclovir or valacyclovir (and probably famciclovir, although this agent is not yet approved by the U.S. Food and Drug Administration for this indication) can reduce symptoms, speed healing by several days, and diminish virus shedding, but does not affect the rate of recurrence of the lesions. Topical and intravenous forms of acyclovir are also approved by the U.S. Food and Drug Administration for treating initial episodes of genital herpes; however, these forms are not commonly used. Valganciclovir is a new oral agent that is converted by the body into ganciclovir. It is active against any virus for which acyclovir is useful (and is also effective for cytomegalovirus), but valganciclovir is more toxic and more expensive than acyclovir. Zidovudine is used for treating HIV infection and would not be used as a single drug for post-exposure prophylaxis against HIV. Corticosteroids are not indicated for treating herpes simplex virus infection and may exacerbate the disorder. Empiric treatment for herpesvirus is reasonable for this patient. However, she also requires evaluation for other causes of genital ulcers and for other sexually transmitted diseases.

Infectious Disease Medicine:Question 63 The correct answer is E Educational Objectives Recall the appropriate isolation precautions for an AIDS patient with cytomegalovirus infection. Critique Cytomegalovirus is transmitted by infected body secretions of patients with prior cytomegalovirus infection. Virus shedding in urine, sweat, tears, and saliva can be intermittent and can persist for months to years after clinical infection has resolved. The virus is transmitted only by persistent, prolonged contact with secretions. Therefore, standard precautions are adequate to protect against infection, and contact, droplet, and airborne isolation are not necessary. Infectious Disease Medicine:Question 64 The correct answer is A Educational Objectives Recognize the most common pathogens implicated as causes of travelers diarrhea. Critique Strains of enterotoxigenicEscherichia coli (ETEC) are among the most commonly implicated causes of diarrheal disease in international travelers. Various other enteric bacterial, viral, and parasitic pathogens have also been reported to cause some form of diarrheal syndrome in travelers. Of the potential pathogens included for this case presentation, ETEC is by far the most common organism causing typical travelers diarrhea. Clostridium difficile is not associated with episodes of diarrhea in travelers.Entamoeba histolytica and Strongyloides stercoralis are only rare causes of travelers diarrhea. In addition, the usual clinical presentation of disease due to these pathogens differs markedly from that of ETEC-associated diarrhea. Salmonella enteritidis is a possibility but is a much less common cause of diarrhea. Travelers diarrhea can be prevented by strict attention to food and water precautions. In addition to oral rehydration therapy, medical treatment of travelers diarrhea usually consists of an antimotility agent (such as loperamide) administered in conjunction with a short course of an appropriate antibiotic (usually a fluoroquinolone). In certain areas of the world, antibiotic resistance to fluoroquinolones is increasing. In these situations, a macrolide antibiotic (for example, erythromycin or azithromycin) may be substituted.

Infectious Disease Medicine:Question 65 The correct answer is B Educational Objectives Recall the approach to diagnosing hepatitis in an HIV-infected patient. Critique This patient, a former injection drug user with HIV infection, has elevated serum aminotransferase values. In the United States, the vast majority of hepatitis C infections occur in injection drug users. In addition, 30% to 40% of persons with HIV infection are co-infected with hepatitis C. Given this patients long history of injection drug use, hepatitis C co-infection would be expected. The enzyme immunoassay for hepatitis C antibodies is both sensitive and specific. However, because of his late-stage HIV

infection, the anti-HCV test result may be negative. Qualitative polymerase chain reaction to detect HCV RNA should be done before beginning a more expensive and potentially invasive workup, such as a liver biopsy. The patient does not have chronic hepatitis B infection and is immune to this disorder. Hepatitis G virus seropositivity may be detectable, but there is no clear association between infection and active hepatitis G. In fact, for unknown reasons, hepatitis G coinfection may improve the clinical course of HIV infection. Disseminated Mycobacterium avium complex infection is possible because of his low CD4 cell count, but this infection is often accompanied by fever, night sweats, diarrhea, and weight loss, which this patient does not have. Infectious Disease Medicine:Question 66 The correct answer is B Educational Objectives Recall the treatment of community-acquired pneumonia in a patient with fluoroquinoloneinduced tachyarrhythmia. Critique Fluoroquinolones may elevate the QTc interval and increase the risk for ventricular tachyarrhythmias. These drugs should not be used for patients with known prolongation of the QTc interval, patients with uncorrected hypokalemia, and patients receiving class IA or class Ill antiarrhythmic agents. A large retrospective study of the treatment of community-acquired pneumonia in hospitalized Medicare patients found that either a fluoroquinolone alone or a combination of a non-pseudomonal third-generation cephalosporin (such as ceftriaxone) plus a macrolide was the most effective treatment for community-acquired pneumonia and that a -lactam/ -lactamase inhibitor combination or any combination with an aminoglycoside was detrimental. However, since macrolides can also prolong the QTc interval, especially in women, changing to ceftriaxone alone is likely to be the best option for this patient. Infectious Disease Medicine:Question 67 The correct answer is E Educational Objectives Recognize and treat varicella pneumonia in a pregnant woman. Critique The patient probably has chickenpox (primary varicella) and varicella pneumonia (note that the presence of lesions at multiple stages on the same part of the body is not consistent with smallpox). Pneumonia as a complication of varicella is more common in adults, especially pregnant women. Moreover, the mortality rate from this complication is increased in pregnant women. Intravenous administration of acyclovir is appropriate despite the lack of prospective controlled studies for this indication and concerns about using drugs that interfere with DNA synthesis during pregnancy. Although the appropriate intravenous dose of acyclovir for herpes simplex virus infections is 5 mg/kg every 8 hours, the appropriate dose for varicella infections is 10 mg/kg every 8 hours because varicella-zoster virus is less sensitive than herpes simplex virus to acyclovir. Oseltamivir is used for influenza virus, which would not explain the skin rash. Although ganciclovir would also treat varicella-zoster virus, there is no reason to choose this more toxic and more expensive agent. Oral acyclovir would not achieve the serum levels thought to be necessary for this life-threatening illness. As an aside, administration of valacyclovir, an oral prodrug that is converted to acyclovir, can by itself result in serum levels of acyclovir that approach those obtained with intravenous acyclovir. However, valacyclovir would probably not be used in this setting because of the life-threatening nature of the illness and the greater need for reliable serum drug levels. Infectious Disease Medicine:Question 68 The correct answer is B

Educational Objectives Understand the current recommendations for antibiotic treatment of a patient with febrile neutropenia. Critique The patient requires antibiotic treatment. An oral regimen such as ciprofloxacin plus amoxicillin/clavulanate has been shown to be as good as or better than an intravenous regimen when a patient has an absolute neutrophil count greater than 1 OO4iL, has no evidence of an intravascular catheter infection, has no signs of sepsis, lives with a responsible family member, and lives close to medical facilities. Trimethoprim/sulfamethoxazole has not been shown to be effective in this setting. Vancomycin should not be used as initial therapy unless -lactam-resistant staphylococci are suspected, as may occur with an intravascular catheter infection. Gram-negative organisms are a likely cause of fever in neutropenic patients, and intravenous cefepime alone or combined with an intravenous aminoglycoside has been used effectively when an oral regimen is contraindicated. Infectious Disease Medicine:Question 69 The correct answer is E Educational Objectives Recognize the indications for use of recombinant human activated protein C (aPC or drotrecogin alfa [activated]) and the complications associated with its use. Critique A large double-blind clinical trial recently showed that recombinant human activated protein C (aPC or drotrecogin alfa [activated]) is effective for the treatment of severe sepsis. Patients who benefited most had APACHE II (Acute Physiology and Chronic Health Evaluation II) scores greater than 24 and multiple organ dysfunction. Indications for use of aPC include a high risk of death from underlying sepsis, the presence of systemic inflammatory response syndrome, acute onset of organ failure, and shock. Bleeding is the major risk of therapy with aPC. The patient with Pseudomonas infection should be given this drug, as he is least likely to have bleeding complications following administration. The man with an ischemic bowel is likely to undergo an emergency surgical procedure and is therefore at higher risk for bleeding, as are the woman recovering from coronary artery bypass grafting and the woman about to undergo surgery for a ruptured appendix. The transplant recipient who has thrombocytopenia is also more likely to develop bleeding complications following the use of a drug with anticoagulant properties. Infectious Disease Medicine:Question 70 The correct answer is A Educational Objectives Understand the interpretation and management of tests for HIV infection. Critique This patients presentation is not uncommon. Although routine HIV screening tests by enzyme immunoassay are highly accurate (both sensitive and specific), false-positive results do occur. Western blot is useful for distinguishing some false-positive results, although this test sometimes provides indeterminate results, such as single bands at p24, p55, or both. Such findings likely represent a false-positive result for patients with no HIV risk behavior. However, for patients with risk behavior, such as the young woman described here, the result could mean impending seroconversion. Under such circumstances, a negative plasma HIV RNA viral load is somewhat reassuring, and a very high viral load (> 10,000 copies/mL) indicates acute HIV infection. Low-level viral loads (< 10,000 copies/mL) have erroneously been interpreted as indicating acute HIV infection in individuals who were later found not to be infected. Hence, this patients low result of 375 copies/mL is not really helpful and is potentially misleading. The appropriate interpretation is to consider this an indeterminate result and repeat serologic testing at various intervals for 6 months. If the serologic test results are still

indeterminate, the patient should be considered HIV seronegative. There is no role for repeat viral load testing or CD4 cell count determination in this setting. Infectious Disease Medicine:Question 71 The correct answer is A Educational Objectives Understand the management options for shingles (herpes zoster). Critique This patient has a typical clinical presentation of shingles (herpes zoster). Acyclovir, valacyclovir, and famciclovir are all approved by the U.S. Food and Drug Administration for treatment, although clinical judgment is needed in deciding whether to treat. For example, therapy should be started within 72 hours of the onset of rash, but a longer period is acceptable if the eye is involved. Some experts would reserve treatment for those over 50 years of age or for patients with moderate or severe rash or pain and/or ophthalmologic involvement. Although data are not always consistent, there is consensus that all three drugs can hasten resolution of the rash and may also prevent some cases of postherpetic neuralgia. No data suggest that topical antiviral agents are beneficial for treating shingles. Antibacterial agents are not indicated for treatment of viral infections and should not be used to prevent bacterial superinfections. Intravenous acyclovir would be expected to be at least as effective as oral acyclovir but would not be used in a normal host with herpes zoster. However, many experts would use intravenous acyclovir for an immunocompromised patient with herpes zoster. Despite previous suggestions that corticosteroids alone can prevent postherpetic neuralgia, there now is good evidence that these agents do not provide this benefit. In addition to the antiviral agent, a 3-week course of corticosteroids could be considered, based on a study that showed accelerated healing and improved quality of life (but no decrease in postherpetic neuralgia) when prednisone was added to a 3-week course of acyclovir. However, this study excluded patients with relative contraindications to corticosteroids, such as hypertension or diabetes mellitus. Infectious Disease Medicine:Question 72 The correct answer is E Educational Objectives Know when and how to provide endocarditis prophylaxis prior to a dental procedure for a patient with mitral regurgitation. Critique Persons with Doppler-demonstrated mitral regurgitation or an audible murmur of mitral regurgitation are considered to be at moderate risk for endocarditis and are candidates for dental prophylaxis, although mitral valve prolapse without regurgitation generally is not considered an indication for prophylaxis. The most recent American Heart Association recommendations, published in 1997, include a discussion of these issues. Although not all dental procedures are considered to require prophylaxis in patients at high or moderate risk, dental extractions and other procedures during which bleeding is anticipated are indications for antibiotics. Clindamycin is the first alternative to amoxicillin for a patient with a history of a significant penicillin allergy, although cephalexin or cefadroxil can be used if the allergy is mild. Azithromycin or clarithromycin can also be considered. None of the currently recommended oral prophylactic regimens for dental procedures requires more than one dose of an antibiotic given 1 hour before the procedure. Moreover, giving a cephalosporin to a person with a history of an immediate-type hypersensitivity penicillin reaction is risky. Infectious Disease Medicine:Question 73 The correct answer is D

Educational Objectives Understand the proper uses of amphotericin B derivatives in treating a patient with a severe fungal infection. Critique The development of fever and a new pulmonary infiltrate in the setting of prolonged neutropenia strongly suggests the presence of pulmonary fungal infection. Although knowing which fungus is present is desirable, it is not possible to determine this noninvasively. Even bronchoscopy has limitations in a person with severe thrombocytopenia. In this case, the patient simply refused the procedure. When new pulmonary infiltrates develop in this setting, beginning an antifungal agent with reasonable activity againstAspergillus is a well-established principle. Amphotericin B is the best studied antifungal agent and is usually active in vitro and in vivo againstspergillus species. Four types of amphotericin B are now available: amphotericin B deoxycholate (the conventional form), liposomal amphotericin B, amphotericin B colloidal dispersion, and amphotericin B lipid complex. Each has a different formulation. All are more or less equally effective but vary in their degree of nephrotoxicity and how well they are tolerated by the patient. Liposomal amphotericin B is preferred for this patient because it is the least nephrotoxic formulation and has mild infusion-related side effects. Amphotericin B deoxycholate has a high rate of infusional side effects and is associated with early onset of nephropathy. Amphotericin B colloidal dispersion may work well and will probably cost less, but it has a high incidence of infusional toxicity and is not used extensively in the United States. Fluconazole and flucytosine are not effective against Aspergillus species. Nystatin can inhibit the growth of fungi when given topically, but this preparation is not suited for systemic therapy. Other correct choices would be amphotericin B deoxycholate with careful monitoring for nephrotoxicity or amphotericin B lipid complex with careful monitoring for infusional reactions. Caspofungin, voriconazole, or itraconazole could also be used. However, the experience with these drugs is limited (but promising) to date. Caspofungin has been approved for treating patients withAspergillus infections who have not responded to amphotericin B or for patients who are intolerant of amphotericin B. Voriconazole may become the standard of care for fungal infections if new data confirm the results of a comparative trial with liposomal amphotericin B showing equivalent outcomes with fewer side effects when voriconazole is used. Infectious Disease Medicine:Question 74 The correct answer is E Educational Objectives Understand the appropriate management of antibiotic prophylaxis for opportunistic infections in an HIV-infected patient successfully treated with antiretroviral therapy. Critique Prior to the advent of antiretroviral therapy, the relapse rate for most HIV-related opportunistic infections was sufficiently high that secondary prophylaxis (for example, forPneumocystis carinii pneumonia) or maintenance therapy (for example, for cryptococcal meningitis) was recommended to be continued indefinitely. Since highly active antiretroviral therapy (HAART) has been introduced, the overall incidence of opportunistic infections has decreased dramatically. Furthermore, several studies have shown that the relapse rate for a number of opportunistic infections is extremely low when a patients CD4 cell count has recovered with HAART, whether or not secondary prophylaxis or maintenance therapy has been discontinued. As a result, guidelines for opportunistic infection prophylaxis and management now recommend that long-term therapy can be safely discontinued for the major AIDS-related opportunistic infections, including P. cariniipneumonia, disseminated Mycobacterium avium complex infection, cytomegalovirus retinitis, cryptococcosis, and toxoplasmosis. This patients prophylactic medications (valganciclovir, trimethoprim/sulfamethoxazole,

and azithromycin) can therefore be stopped. His antiretroviral regimen should not be reduced to a two-drug regimen because of the known risk of viral rebound. Infectious Disease Medicine:Question 75 The correct answer is A Educational Objectives Recall the appropriate management of a patient with vancomycin-resistant enterococci. Critique All patients with vancomycin-resistant enterococci have colonization of the gastrointestinal tract. There is currently no way to eradicate this carriage. What is important is to try to prevent spread of this organism to other patients. Since organisms are often transmitted from the hands of health care workers, careful attention to use of gloves, hand washing, and other means of preventing spread are best implemented by contact isolation. Oral vancomycin has no role in treating or preventing vancomycin-resistant enterococci and may actually be a predisposing factor. Although vancomycin may be reasonable for treating Clostridium difficile, the patients clinical findings are not consistent withC. difficile infection. Imipenem is not active against most strains of vancomycinresistantEnterococcus faecium. Intestinal leaks can occur after bowel surgery, but this patient has no features of peritonitis or other indications of bowel leakage. Infectious Disease Medicine:Question 76 The correct answer is A Educational Objectives Understand infectious complications that develop in an immunosuppressed patient. Critique Adenovirus infection may occur in as many as 20% of patients following stem cell transplantation, and as many as 50% of these infections are fatal. Hemorrhagic cystitis and pneumonia are both common manifestations of adenoviral infection in this setting. The other organisms listed are less likely to have caused this patients findings. Although cytomegalovirus, respiratory syncytial virus, and influenza virus may cause pneumonia in this setting, they are unlikely to be associated with hemorrhagic cystitis. Epstein-Barr virus would not be expected to cause either hemorrhagic cystitis or pneumonia. Infectious Disease Medicine:Question 77 The correct answer is B Educational Objectives Recognize the mechanisms underlying cardiovascular dysfunction secondary to severe sepsis. Critique Shock is defined as a systolic blood pressure of less than 90 mm Hg that is not corrected by fluid resuscitation or that necessitates use of vasoactive agents. Patients with shock caused by severe sepsis often present with hypovolemia due to poor oral intake, gastrointestinal losses, and/or insensitive fluid losses (caused by fever, tachycardia, and increased respiration rate). Initial cardiac parameters reveal a low cardiac index, normal or increased systemic vascular resistance, and low pulmonary capillary wedge pressure associated with volume depletion. Once intravascular volume is restored, persistent hypotension is frequently the result of decreased systemic vascular resistance, which may be due to the effects of lactic acidosis, toxins from microorganisms, or inflammatory mediators. The antibiotics that this patient is receiving are appropriate for the empiric treatment of community-acquired pneumonia and would be expected to be effective against the likely causative organisms. The high cardiac index associated with sepsis would not contribute to hypotension. Poor oxygenation is unlikely, since the patient was intubated and is being mechanically ventilated. In addition, poor oxygenation alone is not an explanation for the

hypotension. In a patient who has received adequate volume replacement, the pulmonary capillary wedge pressure would be normal (or high if cardiac contractility is significantly impaired in association with cardiac failure) and would not correlate with the presence of hypotension. Infectious Disease Medicine:Question 78 The correct answer is D Educational Objectives Recall the treatment of pulmonary reactivation tuberculosis. Critique This patient has a classic presentation of reactivation tuberculosis. He is homeless and has spent time in shelters, which have been sites for transmission of tuberculosis, particularly from the late 1 980s through the early 1 990s, when the incidence of tuberculosis in the United States was on the rise. The possibility of tuberculosis was recognized early in his hospital course, and he was placed in respiratory isolation to protect hospital staff members. Although his initial presentation could have been due to aspiration pneumonia, the positive smear for acid-fast bacilli is strongly suggestive of reactivation tuberculosis. Empiric antituberculous therapy should be started without waiting for culture and sensitivity results. Daily isoniazid, rifampin, pyrazinamide, and ethambutol for 2 weeks, followed by the same drugs twice weekly by directly observed therapy for 6 weeks, followed by isoniazid and rifampin twice weekly by directly observed therapy for 16 weeks is one of three recommended regimens for the management of tuberculosis when drug resistance is neither known nor suspected. The alternative regimens are 1) isoniazid, rifampin, pyrazinamide, and ethambutol or streptomycin three times weekly for 24 weeks by directly observed therapy; and 2) daily isoniazid, rifampin, pyrazinamide, and ethambutol or streptomycin for 8 weeks followed by daily, twice, or thrice weekly isoniazid and rifampin (either daily or twice weekly or three times weekly) for 16 weeks by directly observed therapy. Two-drug regimens, such as isoniazid and rifampin daily for 36 weeks, are no longer recommended. A three-drug ethambutol-containing regimen is not recommended unless the patient has definite intolerance for or a contraindication to administration of pyrazinamide. Finally, unless drug resistance is documented, a five-drug, 24-week regimen is unnecessarily complex and likely to be considerably more toxic. Whichever regimen is chosen for this patient, given his vagrancy, directly observed therapy utilizing a twice or three times weekly regimen should be employed for most of his treatment course. Infectious Disease Medicine:Question 79 The correct answer is B Educational Objectives Understand the principles of antiretroviral management for a patient who fails to benefit from therapy. Critique This case exemplifies some of the difficulties of antiretroviral drug management. In clinical studies of antiretroviral therapy, data consistently show that 60% to 80% of treatment-nave patients achieve an undetectable HIV RNA viral load by 6 months. The response rate for patients in clinical practice is substantially lower, primarily because of poor adherence. This patient appears to have followed her treatment regimen but has had a suboptimal response. Most experts would change the therapy at this time. In order to optimize this change, a resistance assay should be performed (genotype or phenotype) to learn which resistance pattern has developed and choose the most appropriate treatment regimen based on this pattern. Short-term virologic outcomes have been shown to be superior when the results of resistance testing are used to assist in decisionmaking regarding antiretroviral regimen changes. There is no real role for a CD4 cell count or HIV RNA viral load determination for this

patient. Her counts have changed very little over the 7 months of treatment. In addition, progressive immunologic or clinical decline would be an indication for a treatment change, regardless of the viral load response. Switching from nevirapine to efavirenz is not indicated, since resistance test results are not known and making single-drug substitutions is generally not advisable. Moreover, since a single base substitution at the 103 position of reverse transcriptase renders the virus resistant to all non-nucleoside reverse transcriptase inhibitors, the change from nevirapine to efavirenz would not be helpful. Infectious Disease Medicine:Question 80 The correct answer is B Educational Objectives Understand the current recommendations for antibiotic treatment of a patient with febrile neutropenia. Critique This patient has been profoundly neutropenic for a long time. Her infection did not improve while she was on an adequate antibacterial regimen, and a fungal infection should therefore be considered. Candida and Aspergillus, as well as other fungi, require empiric treatment in this setting. Amphotericin B should be started even though a fungal infection has not been identified at this time. Cefepime and gentamicin should not be stopped, especially if the fevers continue without an etiologic agent being identified. Ciprofloxacin is unlikely to be effective against causative organisms that are not covered by the current therapeutic regimen. Making no change in the therapeutic regimen is incorrect because amphotericin B should be added. These recommendations may change in the future as new antifungal agents become available. Infectious Disease Medicine:Question 81 The correct answer is E Educational Objectives Recall the treatment of a patient with pneumonic tularemia. Critique Although the pneumonic form of tularemia may occur naturally, the possibility of a bioterrorism event must be considered when this infection is encountered. Treatment is the same regardless of the cause. Gentamicin (and streptomycin) are the preferred firstline antibiotic agents because of many years of experience using these drugs for treatment of all forms of tularemia. Chloramphenicol, ciprofloxacin, and the tetracyclines are second-line drugs for treating tularemia. Ceftriaxone is not considered effective for this disease. (It is worth noting that landscapers are more susceptible to acquiring pulmonic tularemia because of their exposure to aerosols contaminated with portions of animal carcasses or excretions of infected animals.) Infectious Disease Medicine:Question 82 The correct answer is C Educational Objectives Understand the treatment of cytomegalovirus retinitis in a patient with AIDS. Critique Cytomegalovirus (CMV) retinitis in patients with AIDS usually responds to ganciclovir, foscarnet, or cidofovir. The choice of drug is guided by the need to avoid the significant toxicities of these agents in a specific patient. Ganciclovir is more toxic to the bone marrow, whereas foscarnet and cidofovir are more nephrotoxic. This patient should be treated with ganciclovir because he has better bone marrow reserve than renal function. Valganciclovir, an oral prodrug of ganciclovir, was recently approved by the U.S. Food and Drug Administration for treatment of CMV retinitis in AIDS patients and provides a more convenient option for this patient. Evaluation should also begin for an appropriate

antiretroviral drug regimen. Relapse can be expected, however, unless he receives an anti-CMV maintenance drug regimen or his immune status improves with antiretroviral therapy. Direct delivery of an anti-CMV drug into the eye is an option for patients with CMV retinitis. However, intravitreal injection would not protect this patients currently uninvolved eye, which is an important consideration in such a severely immunocompromised patient. A regimen of highly active antiretroviral therapy (HAART) will take time to increase the CD4 cell count, but this patient is at risk of losing his sight before that happens. Any treatment plan must therefore include immediate use of an anti-CMV drug. Acyclovir has insufficient in vitro activity against CMV to be useful. Finally, there is no role for corticosteroids in the treatment of CMV retinitis. Infectious Disease Medicine:Question 83 Correct answer is B Educational Objectives Recall the appropriate management of hepatitis B exposure in a health care worker. Critique Of the three most common viral bloodborne pathogens (hepatitis B, hepatitis C, and HIV), hepatitis B is the easiest to acquire via percutaneous exposure in the absence of immunity. This nurse may be immune to hepatitis B because of his prior vaccinations, but his status is unknown at the time of the exposure. Confirmation of the immune status of health care workers with an unknown response to vaccination is required at the time of the exposure. Therefore, the first thing to do is to check his immunity. Because postexposure hepatitis B prophylaxis is effective if administered within the first 72 hours after exposure, there is time to wait for the results of immunity testing. If the nurse is not immune, he will require hepatitis B immune globulin and his vaccination series should be restarted. If he is found to have adequate immunity, no prophylaxis is required. Infectious Disease Medicine:Question 84 The correct answer is B Educational Objectives Recognize an adverse reaction to linezolid administration. Critique This patients thrombocytopenia and granulocytopenia are probably secondary to an adverse reaction to linezolid. Thrombocytopenia develops in about 3% of patients who receive linezolid, and granulocytopenia is noted even less frequently in these patients. These findings usually occur after 2 weeks of therapy but may develop earlier and are reversible after discontinuing the drug. For this reason, the manufacturer recommends that all patients receiving linezolid have a complete blood count each week. Disseminated intravascular coagulation usually develops in patients with sepsis and uncontrolled infection. Hypersensitivity vasculitis may be accompanied by other manifestations of an allergic reaction, such as fever, malaise, polyarthralgias, urticaria, and eosinophilia. Thrombotic thrombocytopenic purpura is usually associated with evidence of intravascular hemolysis, neurologic events, and renal failure.

Infectious Disease Medicine:Question 85 The correct answer is D Educational Objectives Select the appropriate empiric antimicrobial regimen for a patient with purulent

meningitis after neurosurgery. Critique


This patient developed meningitis following surgical repair of a leaking aneurysm and has a ventriculostomy tube in place postoperatively. Although the patient could have chemical meningitis, based on the CSF findings of elevated neutrophils, low glucose, and elevated protein, an empiric antimicrobial regimen should be initiated pending culture results. The most likely causative organisms are staphylococci (both Staphylococcus aureus and coagulasenegative staphylococci) and gram-negative bacilli (including Pseudomonas aeruginosa). Vancomycin plus ceftazidime is the only regimen listed that will provide adequate coverage against these organisms and should be initiated pending culture results.

Infectious Disease Medicine:Question 86 The correct answer is A Educational Objectives Recall the diagnosis of Rocky Mountain spotted fever. Critique The patient presents with fevers, chills, and a petechial rash, which are characteristic of a rickettsial infection. His symptoms developed during the endemic season (April through September) for Rocky Mountain spotted fever, and he resides in North Carolina, which is a highly endemic area for this infection. Meningococcal infection can produce similar symptoms. However, the petechial rash is usually on the trunk and is not limited to the wrists. Lyme disease does not usually cause septic findings, and Babesia infection rarely produces a petechial rash. Infectious Disease Medicine:Question 87 The correct answer is C Educational Objectives Recognize the indications for caspofungin for a patient with a refractory fungal infection and intolerance to amphotericin B. Critique The management of fever in a neutropenic patient has been so well studied that death from infectious complications is now rare. In addition to careful clinical evaluation, each patient with febrile neutropenia requires appropriate cultures and other laboratory tests followed by administration of suitable antibiotics. The use of antifungal agents is not indicated for most neutropenic patients at the first sign of fever. However, the failure to respond to antibacterial agents or a breakthrough fever while receiving antibiotics is an indication for administration of antifungal drugs. Traditionally, amphotericin B (or one of its lipid formulations) is the drug of choice. These agents have been studied the longest and are usually fairly well tolerated, although they may cause serious adverse effects. However, this patient had an allergic reaction to amphotericin B. Allergic reactions to this drug are uncommon, but not unheard of. All amphotericin formulations contain the same underlying drug, although they have varying rates of adverse reactions. A true allergic reaction to one formulation should be considered a reaction to all. Because of her allergic reaction, the patient should be started on caspofungin, which has recently been approved for patients who cannot tolerate amphotericin B. Voriconazole shows good efficacy and safety for treatment of febrile neutropenia. However, this patient has two risk factors that make voriconazole a less attractive choice. First, the prior use of fluconazole may select for relatively voriconazole-resistant fungi. Second, and more importantly, voriconazole may induce hallucinations, which are usually mild and self-limited. Nevertheless, the patients recent history of hallucinations may make this possible side effect especially unsettling for her. Infectious Disease Medicine:Question 88 The correct answer is A Educational Objectives

Recall that mild sepsis does not require recombinant human activated protein C (aPC or drotrecogin alfa [activated]) therapy. Critique This patients rapid decline is somewhat confusing at this early time. The most likely causes relate to infection, oversedation, transient hypotension, and lung injury. Given the extensive differential diagnosis, the initial steps should include providing supportive care, beginning treatment of the most serious disorders, and continuing the evaluation. The degree of anticoagulation is adequate and perhaps somewhat higher than adequate. However, reversing the anticoagulation with vitamin K is not needed, since the patient has no evidence of bleeding but does have atrial fibrillation, which carries a high risk for embolic disease. Nonetheless, a pulmonary embolism is unlikely at this time because the patient has only diffuse chest radiographic changes and a therapeutic INR. Therefore, a vena cava filter is not indicated. Antifungal therapy is unlikely to be beneficial, since fungal infections are seldom encountered as nosocomial processes in this patient population, and there are no indications of a primary fungal disease. Although recombinant human activated protein C (aPC or drotrecogin alfa [activated]) would seem to be potentially useful, it should not be given for two reasons. First, the elevated INR may increase the risk of bleeding (which is increased by the intrinsic activity of aPC). Second, the illness does not yet seem to involve multiple organs. There is no advantage to administering aPC therapy, especially because of the risks and cost of this agent. Some clinicians use an APACHE II (Acute Physiology and Chronic Health Evaluation II) score of greater than 24 to decide about suitability of aPC in someone with suspected or documented sepsis, but this patient is not yet that ill. Infectious Disease Medicine:Question 89 The correct answer is B Educational Objectives Be aware of the effects of increased fluoroquinolone-resistant gonorrhea on the treatment of this disorder. Critique Despite the patients self-treatment with ciprofloxacin, he had a poor clinical response most likely because of fluoroquinolone-resistant Neisseria gonorrhoeae. Resistance of N. gonorrhoeae strains to fluoroquinolones is an increasing problem. In some cases, this resistance is marginal, and a good treatment outcome can be achieved by increasing the dose. However, some strains are sufficiently resistant so that fluoroquinolone therapy is ineffective. The normal course of ciprofloxacin treatment for gonorrhea is very brief (usually a single dose). Therefore, underdosing is unlikely to explain this treatment failure. Although the patient may also have acquired syphilis during his travels, this should not interfere with treatment of gonorrhea. Absorption of oral ciprofloxacin is usually very good, with bioavailability of 80% or more. Drugs that interfere with absorption of fluoroquinolones include divalent cations such as calcium and magnesium that are most commonly taken as antacids or food supplements. The histamine blocker that he was taking should not interfere with the bioavailability of ciprofloxacin.

Infectious Disease Medicine:Question 90 The correct answer is D Educational Objectives

Understand the management of an HIV-infected patient presenting with P. carinii pneumonia. Critique This is a typical presentation ofPneumocystis carinii pneumonia in a patient with newly diagnosed late-stage HIV infection. The key to management is recognizing that his hypoxemia at the time of hospital admission indicative of a poor prognosis. The patient was appropriately diagnosed as HIV positive, his classic chest radiographic findings were correctly interpreted as likely to be due toP. carinii pneumonia, and he was appropriately treated with high-dose parenteral trimethoprim/sulfamethoxazole. However, because of his low arterial blood Po2 at the time when P. carinii pneumonia was diagnosed, he should also have received adjunctive corticosteroid therapy. Corticosteroids have been shown to improve both morbidity and mortality in this setting. Changing antibiotics or adding an additional antibiotic forP. carinii pneumonia might be considered later in his course, if he fails to benefit from the current antibiotic regimen and the addition of corticosteroid therapy. Although patients with sulfonamide-resistant P. carinii pneumonia have been described, the management of such patients has never been adequately established. The value of starting highly active antiretroviral therapy in an AIDS patient who has an acute opportunistic infection is also controversial. Although such treatment may induce early beneficial immune reconstitution, it could conceivably worsen the inflammatory response and thereby worsen this patients respiratory injury. Finally, the addition of azithromycin to cover atypical pathogens would not be important at this time, since the diagnosis is known and since prednisone was not started earlier. Infectious Disease Medicine:Question 91 The correct answer is D Educational Objectives Consider malaria as a potential diagnosis in all ill travelers returning from a malaria-risk area. Critique The first consideration in the evaluation of this patient is to determine whether his itinerary could have placed him at risk for the development of malaria. His time in rural areas of Cambodia and Vietnam does indeed represent a significant exposure risk. He was taking no prescription medications, including malaria prophylaxis. Because malaria is potentially fatal in a nonimmune host, the most important first step in this patients management is to establish or exclude this diagnosis. In addition to the standard tests and cultures indicated for the workup of an acutely febrile patient, thick and thin blood smears should be examined. Other causes of fever should also be considered. In the case of a moderately to severely ill individual in whom no definitive alternative diagnosis can be made, it may be prudent to begin empiric antimalarial treatment while waiting for the blood smear results. Repeating the malarial blood smears several times over the next few days may be necessary if the initial smears are negative but the patient remains ill. For travelers at risk, most cases of malaria can effectively be prevented by the use of anti-mosquito personal protection measures in combination with appropriate chemoprophylaxis. The choice of the most appropriate chemoprophylactic agent is based on the travelers exact itinerary, length of stay, and personal medical profile. Hepatitis B, yellow fever, and an amebic liver abscess are not the first considerations in diagnosing this patients illness.

Infectious Disease Medicine:Question 92 The correct answer is C Educational Objectives

Recall interventions to prevent ventilator-associated pneumonia. Critique Prevention of infections in an intensive care unit (ICU) is difficult. There are relatively few proven interventions to help reduce the risk of infection in an IOU. However, there are interventions for preventing ventilator-associated pneumonia. Even when a cuffed tube is in place, bacteria from the stomach can reach the lungs and cause pneumonia. Semierect positioning in bed is useful because it prevents the excursion of bacteria from the stomach into the upper airways. In many patients in ICUs, use of I-Q-receptor blockers permits a high density of bacteria in the stomach. These organisms are frequently pathogenic and resistant to standard antibiotics. Reducing the density of gastric bacteria by use of prophylactic antibiotics is tempting. However, this approach is ineffective and serves to select for even more resistant strains. Oral placement of endotracheal tubes is currently believed to be superior to nasal placement because nasogastric and nasotracheal tubes cause some degree of obstruction of the ostia in the nose, which can predispose to nosocomial sinusitis. Whether all nasal tubes should be replaced by oral tubes is unclear. However, no benefit will be gained by changing from an orotracheal to a nasotracheal tube. Changing endotracheal tubes seems logical, but reintubation is associated with certain risks (intubating the esophagus, precipitating hypoxia during the procedure, and so on). Reintubation may also increase the risk of nosocomial pneumonia. Careful inspection and management of the tubing can help reduce infections slightly. Because the tubing has a tendency to collect water, careful drainage of accumulated condensate into patientspecific drainage containers is advocated. Cooling blankets are often used to control body temperature in patients who are critically ill. However, there is little support for this measure. In addition, cooling blankets are uncomfortable for awake patients and do not prevent complications related to infections. Infectious Disease Medicine:Question 93 The correct answer is C Educational Objectives Recognize the clinical presentation and recall the long-term treatment of a patient with chronic granulomatous disease. Critique This patient has chronic granulomatous disease, as demonstrated by the clinical course and the abnormal nitroblue tetrazolium test. Such patients commonly develop infections due to Staphylococcus aureus and other catalase-positive organisms. Patients with chronic granulomatous disease have been shown to have fewer infections when given prophylactic antibiotics, with or without interferony. Therefore, treatment is indicated. Several studies show that trimethoprim/sulfamethoxazole plus interferon- is the most effective regimen, and this regimen is now the standard of care. Dicloxacillin is not indicated, as penicillins and other -lactam antibiotics are not as effective as other antibiotic agents, presumably because of ineffective intracellular concentrations of these antibiotics. Low-dose prednisone and interleukin-2 are not correct because corticosteroids and interleukin-2 have not been shown to be effective when given as long-term prophylaxis to a patient with chronic granulomatous disease.

Infectious Disease Medicine:Question 94 The correct answer is C Educational Objectives

Understand the treatment of recurrent genital herpesvirus infections. Critique The patient appears to be having the prodrome of recurrent genital herpes, at which time the institution of antiviral therapy is most beneficial. Therefore, starting oral acyclovir, valacyclovir, or famciclovir is appropriate. Although 5 days has been the standard duration for treatment of recurrent episodes (compared with 10 days for initial episodes), new data suggest that even shorter regimens may be used for recurrences. Each of these agents can also be used to suppress frequent recurrences of genital herpes, although episodic treatment may be more appropriate for this patient who has only two to three episodes each year and a recognizable prodrome. Although topical acyclovir is approved by the U.S. Food and Drug Administration for treating initial episodes of genital herpes, it is less useful for recurrences. Chronic suppressive therapy is typically used for patients with at least six recurrences yearly. Finally, corticosteroids are not indicated for herpes simplex virus infections and could exacerbate the condition.

Infectious Disease Medicine:Question 95 The correct answer is A Educational Objectives Understand the widespread prevalence of Chlamydia infection in young women. Critique Several infectious diseases are caused byChlamydia. Infection of the lungs can occur with C. psittaci or C. pneumoniae. Infection of the eye can occur withC. trachomatis. This last species also causes genital infections, which seem to be especially common in young people. Most screening programs have been directed towards young women, although young men also seem to have a high rate of positive test results. The older system of detecting Chlamydia utilized cultures, which were slow and insensitive. However, more recent tests, such as ligase chain reaction, are quicker, easier to perform, and more sensitive than cultures. As a result, we now are more aware of how common asymptomatic Chlamydia infections can be. Curiously, these infections seem to be less common in older womenperhaps as a result of changes in the genital mucosa. There is no evidence that genital Chlamydia infections occur in animals or are transmitted in ways that do not involve genital contact.

Infectious Disease Medicine:Question 96 The correct answer is D Educational Objectives Recall appropriate isolation precautions for a patient with herpes zoster. Critique Varicella-zoster virus can be transmitted from patients with shingles by both airborne transmission and direct contact. Therefore, both airborne and contact isolation are required. Droplet isolation is not indicated, and standard precautions, contact isolation, or airborne isolation alone would be inadequate.

Infectious Disease Medicine:Question 97 The correct answer is D Educational Objectives

Recognize severe complications of nucleoside analogue treatment in an HIV-infected patient. Critique This patient presents with a rare complication of nucleoside analogue treatment, lactic acidosis/hepatic steatosis. This complication has most often been associated with the dideoxynucleotide agents, especially stavudine and didanosine, both of which this patient is taking. However, all the agents in this class have the potential to cause this syndrome. The mechanism of injury is likely to be mitochondrial in nature. Imaging studies of the abdomen will usually demonstrate fatty infiltration of the liver. Lactic acidosis/hepatic steatosis has a high fatality rate, especially when a patient has very high lactic acid levels. Treatment involves discontinuing the suspected agent(s). This patients entire antiretroviral regimen should be discontinued. Suggested therapeutic measures have included the administration of riboflavin and 1-carnitine. Lactic acidosis resolves slowly over weeks to months. Acute bacterial sepsis is a possible diagnosis but would not account for all the findings, particularly the fatty liver infiltration. Hepatitis, due to either viral agents or alcohol, also would not explain all the findings. Finally, the patient does not appear to be in shock.

Vous aimerez peut-être aussi